Nothing Special   »   [go: up one dir, main page]

Medical-Surgical 1 Nursing Samplex: Exam Compilation

Download as pdf or txt
Download as pdf or txt
You are on page 1of 75

MEDICAL-SURGICAL 1 NURSING SAMPLEX

Exam Compilation

UNIT EXAM 1 (MEDICAL-SURGICAL 1)

FLUIDS AND ELECTROLYTES

1. Mang Jose had an ABG drawn. The result are pH: 7.34, PaCO2: 37mmHg , PaO2:79mmHg, HCO3: 19mEq/L The
nurse interprets that Mang Jose is experiencing :
A. Metabolic alkalosis
B. Respiratory alkalosis
C. Metabolic Acidosis
D. Respiratory Acidosis

2. The ABG of Patient George Cy are as follows pH: 7.45, PCO2:30mmgHg, HCO3: 22mEq/L. These results indicate
which condition?
A. Respiratory Alkalosis, Compensated
B. Metabolic Alkalosis, Uncompensated
C. Respiratory Acidosis, Uncompensated
D. Metabolic Acidosis, Compensated

3. Prior in giving Ammonium Chloride a treatment for chloride deficit , the nurse must:
A. Ask if patient is allergy to shellfish
B. Determine the urine output
C. Know the chloride level of the patient
D. Get the vital signs

4. Sodium is both an electrolyte and mineral. Which of the following foods is a good source of sodium?
A. Carrots
B. Cured Ham
C. Jell-O
D. Gummy bears

5. The ABG of a coma patient is pH 7.50, PCO2 of 30 mmHg. The nurse defines that the patient is experiencing
Respiratory Alkalosis. Which laboratory confirms this condition?
A. Phosphorus of 4.o mEq/L
B. Potassium 3.0 mEq/L
C. Magnesium 2.0 mEq/L
D. Sodium of 144 mEq/L

6. Upon assessing the patient with heart failure, the neck veins are distended and the patient is dyspneic. What action
should you do?
A. Increased the drop rate of the IVF and auscultate the heart sound
B. Discontinue the Iv immediately
C. Place the patient on the fowlers position and monitor v/s every 15 minutes
D. Position on a semi-FOwler and prepare the PRN diuretics as ordered

7. Which is the function of Chloride.


A. Sending and receiving nerve signals
B. Supports a healthy immune system
C. Works as a buffer system
D. Regulates body temperature

8. Electrolytes are chemicals that conduct electricity when mixed with water. Most abundant… [INCOMPLETE]
A. Sodium
B. Potassium
C. Magnesium
D. Chloride

9. During progressive stages of anaerobic metabolism, the nurse must be aware… [INCOMPLETE]
A. Respiratory Acidosis
B. Respiratory alkalosis
C. Metabolic acidosis
D. Metabolic alkalosis
10. You are caring for a group of patients, which of the following groups is at high risk of dehydration?
A. An infant with diarrhea
B. A 22 year old model who is suffering from mild anorexia
C. An 18 year old basketball player with stress fracture on thee right foot
D. A 45 year old with severe flu

11. The patient lab result shows a slight decrease in Potassium, which of the following foods would you… [INCOMPLETE]
A. Banana
B. Green mango
C. Dates
D. Apples

12. Magnesium plays many crucial roles in the body. Magnesium is absorbed in?
A. Gallbladder
B. Liver
C. Small Intestine
D. Stomach

13 You are assessing a patient with sodium deficient, which of the following are manifestations of hyponatremia
A. Dry skin
B. Orthostatic hypotension
C. Hyperactive muscles
D. Tachycardia
E. Nausea
F. Increased LOC

14. Patient H is recovering from parathyroidectomy. The morning lab results are back. Which of the following…
[INCOMPLETE]
A. Calcium 6.0
B. Calcium 8.8
C. Calcium 9.2
D. Calcium 12.6

15. You are preparing to care for a patient deficient in Potassium. You review the pa.. [INCOMPLETE]
A. Is taking a potassium-sparing Diuretics
B. Has Renal Failure
C. Has a history of Addison’s disease
D. Requires nasogastric suctioning (ge mention ni sir)

16. A Trousseau Sign is best defined as:


A. Sudden, shar pain
B. Involuntary contraction of the muscles in the hand and wrist
C. Numbness around the mouth
D. Twitch of the facial muscles

17. Patient G a bedridden male patient admitted and on the lab result he has a calcium level of 13. 5. Which medication
should be given:
A. 10% Calcium Gluconate
B. Hydrochlorothiazide
C. Calcitonin
D. Calcium Chloride

18. You are caring for a post thyroidectomy patient at risk for hypocalcemia. What action should you take when….
[INCOMPLETE]
A. Evaluate the quality of the patient’s voice postoperatively, noting any drastic changes.
B. Observe swelling of the neck, tracheal deviation and severe pain.
C. Monitor laboratory values daily for an elevated thyroid stimulating hormone
D. Observe for muscle twitching and numbness or tingling of the lips, fingers and toes.

19. You are aware that hyperventilating can lead to which acid and base imbalances:
A. Respiratory Acidosis
B. Metabolic Alkalosis
C. Respiratory Alkalosis
D. Increased PACO2

20. Which is true about Sodium Deficit: SELECT THE CORRECT RESPONSE(S)
A. Hyponatremia can be superimposed by Fluid deficit or excess
B. Sodium may be remove by diarrhea and a problem with Aldosterone
C. Refers to a low serum sodium in the human body
D. Pulse rate of the patient will increase (pwede sad ni)

21. You are caring for a patient placed on NGT and naso gastric irrigation once eve… to use for irrigation?
[INCOMPLETE]
A. Tap water
B. Sterile Water
C. Distilled water
D. Sodium Chloride

22. A patient with Hypocalcemia is on seizure precaution the nurse must:


A. Mayo scissors should be at the side table
B. Suction machine should be at the bedside
C. Position the patient in fowlers
D. Rails the side rails

23. Which of the following is TRUE about isotonic solution:


A. Only solution that can be administered with a blood by product
B. Provides Sodium, Chloride and Free Water
C. Supplies no Calories
D. Treatment for Hypovolemia and early shock

24. You are caring for a patient with Congestive Heart Failure. On Assessment, you notice that the patient is dyspneic and
upon auscultation crackles are audible. What additional signs would you expect to note?
A. Weight Loss (weight gain dapat)
B. An Increased BP
C. Flat Neck and Hand Veins (distended dapat)
D. A decreased Central Venous Pressure (increased dapat)

25. The ABG of Ph 7.26, PACO:28 , HCO3 of 11 mEq/L. How would you interpret this result?
A. Respiratory Acidosis with no compensation
B. Metabolic Acidosis with compensation (partial compensation)
C. Metabolic Alkalosis with compensation
D. Respiratory Alkalosis with no compensation

26. Which of the following shows effective instructions by the nurse to the patients recovering from a HYPERVOLEMIA
problems:
A. Patient uses Amiloride once a day only
B. Patient is minimizing the protein the diet
C. Patient uses lemon for seasoning
D. Patient uses a total of 30 gram of salt daily

27. A newly registered nurse is giving potassium chloride IV as prescribed to a patient which of the following is part of plan
for preparation and administration of the potassium
A. Preparing the medication for bolus administration
B. Monitoring the output during administration
C. Diluting in appropriate amount of Normal Saline
D. Obtaining an IV infusion pump

28. The nurse in the ICU receives arterial blood gasses with a patient who is campla… [INCOMPLETE]
A. Respiratory alkalosis
B. Metabolic acidosis
C. Metabolic alkalosis
D. Respiratory acidosis

29. The sodium level of your patient is 150mEq/L abdb advised for dietary modifica.. [INCOMPLETE]
A. Low Fat Yogurt
B. Peas
C. Processed Oat Cereals
D. Cauliflower

30. You are assessing skin turgor in an elderly patient you should remember that:
A. Dehydration causes the skin to appear edematous and spongy
B. Normal skin turgor is moist and boggy
C. Overhydration causes the skin to tent
D. BASIN D
31. Patient Rose has 3 days of history of nausea and vomiting. Patient Rose is hypoventilating-.. [INCOMPLETE]
The nurse is expecting to note which of the following?
A. An increased Ph and increased HCO3
B. An increased pH and decreased CO2
C. A decrease in pH and Increased CO2 (depende sa question)
D. A decreased pH and a decreased HCO3

32. Magnesium is an important mineral, playing a role in over 300 enzyme reactions in the human body. Which of the
following food…. a good source of magnesium. [INCOMPLETE]
A. Canned white Tuna (sir cabs)
B. Peanut butter (sir cabs)
C. Peas (sir cabs)
D. Cauliflower (sir cabs)
E. Oranges
F. Bacon

33. The lungs participate in acid and base balance by:


A. Reabsorbing HCO3
B. Sending Hydrogen ions to renal tubules
C. Using the CO2 to regulates Hydrogen ions
D. Splitting carbonic acid into 2

34. Patient Annie Dy has been receiving Torsemide for 2 week now. What electrolytes should you monitor?
A. Potassium
B. Sodium
C. Magnesium
D. Calcium

35. Chloride helps maintain acid and base balance by performing which of the following roles?
A. Participate in chloride shifting
B. Maintaining Balance of cations in ECF and ICF
C. Following the sodium to maintain osmolarity
D. Separating the Carbonic Acid

36. Severe manifestation of hyponatremia would manifest if the level would drop to:
A. 115 mEq/L
B. 130 mEq/L
C. 120 mEq/L
D. 125 mEq

37. The tube to be used for ABG extraction is: (SKIP)


A. White
B. Red
C. Yellow
D. Green

38. Which of the following series of assessment findings indicates RESPIRATORY ALKALOSIS:
A. High serum chloride, low serum potassium and confusion
B. Dizziness and flaccid muscles
C. Low serum potassium, confusion, paresthesia
D. High serum potassium, flaccid muscles and paresthesia

39. Patient Merry Sy is admitted with anemia and is receiving a blood transfusion. Based upon IV solutions may be
administered with blood products?
A. 5% dextrose in water
B. Lactated ringer’s
C. 0.9% NaCl
D. D5 and 45% Normal Saline

40. You are caring for patient who is receiving IV diuretics suspects that the patient is experiencing… [INCOMPLETE]
A. Lung Congestion
B. Decreased Hematocrit
C. Decreased Central venous pressure
D. Increased Blood pressure

41. You are caring for a patient with Congestive heart failure who is receiving a high dose of… suspect Hyponatremia.
What others would you expect to note in this patient if HYPONATREMIA [INCOMPLETE]
A. Decreased urinary output
B. Dry skin
C. Increased urine specific gravity
D. Hyperactive bowel sound (mention ni sir)

42. You are reviewing the patient’s most recent blood gas results indicating pH: 7.43, PCO2 : 31 mmHg, HCO3: 22mEq/L.
You would interpret these results as indicative of?
A. Uncompensated Metabolic Alkalosis
B. Uncompensated Respiratory Alkalosis (either aning duha)
C. Uncompensated Respiratory Alkalosis (Compensated - dapat ang isa ka choice)
D. Compensated Metabolic Acidosis

43. Which of the following is clinical manifestation of a patient with excess in chloride:
A. Bradycardia
B. Lethargy
C. Hypotension
D. Diminished Cognitive Ability
E. Hypervolemia

44. A 65 year old male patient was admitted to a medical-surgical unit 72 hours ago with pyloric….. the nurse taking care
of the patient notices that his potassium is very low and becomes concerned… [INCOMPLETE]
A. Metabolic acidosis
B. Metabolic alkalosis
C. Respiratory acidosis
D. Hypercalcemia

45. ABG of patient China indicates the pH: 7.52, PCO2: 32 mmHg, HCO3: 24 mEq/L. What is the possiti…
[INCOMPLETE]
A. Inadequate nutrition intake
B. Prolonged gastric suction
C. Airway obstruction
D. Excessive mechanical ventilation

46. You are assessing the ECG result of patient Morph and notes that the potassium level is 5.4 meq/l .. following would
you expect to note on the ECG… [INCOMPLETE]
A. Prominent U waves
B. ST depression
C. Inverted T waves
D. Tall peaked T waves (either of the two)

47. The common caused of deficit in Magnesium


A. Drug overused
B. COPD
C. Alcoholism
D. Hepatic encephalopathy

48. Which of the following is not a manifestation of Hypervolemia?


A. Grade II pitting edema
B. Rales
C. Distended Neck veins
D. Anuria

49. Which of the following is a clinical manifestation of a patient with excess in chloride:
A. Bradycardia
B. Lethargy
C. Hypervolemia
D. Hypotension
E. Diminished Cognitive Ability

50. Patient Ireland who is hospitalized with a possible electrolyte imbalance is disoriented and weak pulse, and takes
hydrochlorothiazide. He most likely suffers from:
A. Hypokalemia
B. Hyponatremia
C. Hypernatremia
D. Hyperkalemia

51. All except one is FALSe about Fluid Volume excess;


A. Hemodialysis is treatment may be used to remove waste product and maintain acid and base balance
B. Sodium is not restricted but minimize
C. Proteinuria can occur with FVE
D. A patient can manifest hypotension

52. You are caring for a group of patient reviews electrolytes laboratory results and notes a … mEq/L on one of the
patient’s laboratory reports. You understand that which patient is at high … development of a potassium value at this level
[INCOMPLETE]
A. Patient who sustained a traumatic brain injury (depende sa question)
B. Patient with cushing syndrome
C. Patient who has been overusing laxatives - can cause loss of potassium in the stool
D. Patient with coma

53. You are caring for a patient with Congestive Heart Failure who is receiving a high dose of diuretics. On assessment
you note that the patient's neck has flat veins, generalized muscle weakness, diminished DTR. you suspect hyponatremia.
What other would you expect to note in this patient if Hyponatremia were present?
A. Dry skin
B. Decreased Urinary Output
C. Increased Urine Specific Gravity
D. Hyperactive bowel sound

54. You are assessing an elderly with deficit in Fluid the ebay area to assess the skin turgor is:
A. Arms
B. Thigh
C. Back of the Hand
D. Sternum

55. Patient Corona is discharged from the hospital after having an episode of heart failure. She’s prescribed doses of
digoxin (lanoxin) and furosemide (Lasix). Two days later, she tells the nurse that she feels weak, heart “flutters”. What
action should the nurse take?
A. Tell the patient to rest more often
B. Call the physician, report the symptoms and request to draw a blood sample to determine the patient’s
potassium level
C. Tell the patient to stop worrying
D. Tell the patient to stop taking digoxin

56. The recommended grams of sodium in a day for patient with Hypervolemia is
A. Maximum of 25 grams in a day
B. Maximum of 15 grams a day
C. Maximum of 30 grams in a day
D. MAximum of 20 grams in a day

57. A newly admitted patient who is in the state of oliguric, the nurse orders a fluid challenge of 100 to 200 mL ..normal
saline solution over 15 minutes. The nurse is aware this invention will help: [INCOMPLETE]
A. Distinguish reduced renal blood flow from decreased renal function
B. Evaluate pituitary gland function
C. Provide an effective treatment for hypertension-induced oliguria
D. Distinguish hyponatremia from hypernatremia

58. Bicarbonate is lost during which condition?


A. Diaphoresis
B. Diarrhea
C. Vomiting
D. Diuresis

59. Which of the following foods should a patient with excess calcium avoid?
A. Sardines
B. Pumpkin
C. Potatoes
D. Mushroom

60. The nurse is admitting a patient with a suspected fluid imbalance. The most sensitive indicator of body.. balance is
A. Serum sodium levels
B. Daily weight
C. Urine Output
D. Blood Pressure

61. Hormone that regulates the osmotic pressure


A. Calcitonin
B. Angiotensin
C. Melatonin
D. Vasopressin

62. A patient who is in renal failure partially loses the ability to regulate changes in pH because the kidneys.
A. Regulate and reabsorb carbonic acid to change and maintain pH
B. Buffer acids through electrolytes charges
C. Regenerate through electrolyte changes
D. Combine carbonic acid and bicarbonate to maintain a stable pH

63. You are evaluating the ABG result for a patient who is receiving supplemental oxygen. Which finding would indicate
that the oxygen level was adequate?
A. Po2-40mmHg
B. Po2-45 mmHg
C. Po2-60mmHg
D. Po2-80mmHg

64. A nurse is giving a patient with low blood pressure a hypertonic solution, which will increase the number of
dissolved particles in his blood, creating pressure for fluids in the tissues to shift into the capillaries and increase the
blood volume. Which of the following is associated by these terms?
A. Osmosis
B. Diffusion
C. Hydrostatic Pressure
D. Active transport

65. Which of the following lab results should you refer to immediately?
Answer: Urea - 30

66. Nurse is aware that a patient with hypokalemia could cause which of the following in the ECG?
A. Shorten PR
B. Inverted T waves
C. Depressed U wave
D. Elevated U wave

67. A patient with Magnesium deficit:


A. Carpal Spasm
B. Cold Clammy skin
C. Negative patellar Reflexes

68. Patient is receiving Torsemide, what electrolyte is affected?


Answer: Potassium

69. You are caring for a patient with congestive chuchhu you..There is crackles are very much audible. What additional
assessments would you expect to note?
ANSWER: Increased blood pressure

70. All except one are factors that would lead to hypervolemia?
Answer: Alzheimer’s disease

71. You are caring for a patient with hypovolemia and you should instruct the patient to?
Answer: Instruct the patient to avoid coffee

72. A newly registered nurse is giving potassium chloride IV as prescribed to a patient with hypokalemia. Senior nurse
would determine that the nurse is unprepared…. [INCOMPLETE]
A. Obtain infusion pump
B. Monitor output during the output (could be/pwede)
C. Preparing the administration for bolus administration
D. Diluting the appropriate amount of normal saline

73. Which of the following patients is at risk for developing calcium deficit?
Answer:
A patient with malignancy
Patient taking lithium
Patient uses excess sunscreen
Patient with hyperparathyroidism
Patient uses antacids over prolonged period of time

74. THE NURSE RECOGNIZES THAT THE PHYSIOLOGIC COMPENSATORY CHUCHU… YOU RE
PERFUSED SWEATING
Elevated temperature
Severe dehydration
Renal retention of hydrogen
Boom

75. YOU ARE PREPARING TO CARE FOR THE PATIENT WITH DEFICIT IN POTASSIUM… YOU ARE REVIEWING
THE RECORDS.. [INCOMPLETE]
Answer:
A. Renal failure
B. Has a history of addison’s disease
C. Taking potassium diuretics
D. Requires NGT suctioning

76. With persistent diarrhea, an infant is.. Which of the following will develop [INCOMPLETE]
Answer: Hypovolemia metabolic acidosis

77. During progressive state of anaerobic metabolism. You would be aware of initially… [INCOMPLETE]
Answer: Metabolic acidosis

78. Hypercalcemic crisis manifestation include [INCOMPLETE]


Answer: Polyuria and thirst

79. You are caring for a patient with Congested Heart Failure who is receiving a high dose of diuretics. On assessment
you note that the patient neck has flat veins, generalized muscle weakness, diminished DTR. you suspect hyponatremia.
What other would you expect to note in this patient if Hyponatremia were present?
Answer: Hyperactive Bowel Sound

80. Patient A is receiving an IV potassium chloride. What nursing action is required? [INCOMPLETE]
Answer: Monitor the injection site for redness

81. Patient 2 is newly diagnosed with diabetes mellitus with Nausea, vomiting. Abg result is 7.2. Which other assessment
finding would you anticipate?
Answer: Weakness, dysrhythmia, Kussmaul’s breathing

82. Upon assessment of a patient with heart failure, the neck vein is distended and dyspneic. What action should you do?
Answer: Position the patient on a semi-fowler position and prepare a medications of diuretics as ordered

83. You are reviewing the lab results of your patient with a sodium level of 130.
Which of the following is at risk of developing sodium deficit?
Answer: Patient who are taking diuretics

84. The calcium level of the patient is 4. You understand which of the following patients would likely… [INCOMPLETE]
Answer: Patient with prolonged bed rest

85. You are aware that hyperventilating can lead to which of the following imbalance?
Answer: Respiratory alkalosis

86. Which is the following changes in a patient would you expect with hypocalcemia?
Answer: Prolonged QT interval

87. A patient who is in renal failure partially loses the ability to regulate changes in pH because the kidneys.
Answer: It regenerates and reabsorbs the bicarbonate to… PH

88. Which of the following foods are high in magnesium? (Select all that apply)
A. Peas
B. Bacon
C. Oranges
D. Cauliflower
E. Peanut Butter
F. Canned Tuna

89. Is a patient with hypokalemia on seizure precaution?


Answer: Raise the side rails

90. You are taking care of a patient 65 years old with renal problem. Finger spasms BP cuff
Answer: Hypocalcemia
PERIOPERATIVE NURSING

1. You are taking care of… best time for splinting and leg exercise… post anesthesia [INCOMPLETE]
Answer: Upon admission of the patient

2. The patient is placed on NPO prior to the surgery… [INCOMPLETE]


Answer: Prevent aspiration

3. You are an operating room nurse and will be assisting the patient. What action will you prioritize in this phase?
Answer: Physiologic monitoring

4. You are an emergency room nurse caring for a 5 year old. The patient needs emergency surgery. How should informed
consent can be exercised in this scenario?
Answer: Surgery should be done and consent should be signed later.

5. You are caring for a patient scheduled for a removal of a tumor. The medicine is to be given on call of the OR. When
would be the best time to administer the medication?
Answer: As soon as after being notified by the OR

6. You are caring for 77 year old female for surgery with a cancer. What will you prioritize?
Answer: Risk for Infection (naa nani sa uboos)

7. Post 2 hrs you pt statr to com apportoorve action [INCOMPLETE]


Aplpy warm blannet and supplemental O2
Take the temo o fteh pt and tsb
Give Notify the physician

8. After having your post laparoscopic surgery. What should you monitor to prevent any fatal complication?
Answer: Shock, abdominal obstruction, pulmonary function

9. Laryngectomy. The patient is very much concerned about chuchu communication. Which of the following should be
included in the teaching plan?
Answer: Refer to an esophageal speech therapist

10. The most essential.. Occurrence of pulmonary embolism.


A. 60 yrs with knee replacement
B. 60 yrs with bacterial pneumonia
C. Woman with severe vaginal bleeding
D. 60 yrs with a history of thrombocytopenia

11. You are caring for a patient with bronchoscopy. What actions should be done to prevent laryngeal edema?
Answer: Position the patient in a semi-fowler’s position

12.Thoracostomy what important assessment you need to observe?


Answer: Expectoration of blood

13. Preoperative assessment on a patient subject for surgery. He drinks approx 3 bottles of red wine. What post op
problem would you anticipate?
Answer: Alcohol withdrawal syndrome 2-4 days after his last drink

14. You are teaching a patient on how to perform deep breathing exercises. What action should be your teaching in this
matter?
Answer: Instruct the patient to take a deep breath through the mouth and take a short period of time to cough

15. When performing a pre admission to a patient with orthopedic surgery. What is the main purpose of pre admission
assessment?
Answer: Verify completion of pre-operative diagnostic testing

16. Use of incentive spirometry [INCOMPLETE]


Answer: Never blow strongly

17. You are discussing for a group of people about pulmonary TB… earliest sign of TB: [INCOMPLETE]
Answer: Cough with mucous

18. Pt wit pn waht action is nec… [INCOMPLETE]


Answer:
Auscultate the breath sounds
Auscultate the bowel sounds
Assessing the chest pain

19. You are caring with pneumonia what is our primary prevention of infection
Answer:
A. Suction airway clearance
B. Position the pt every 2 hours
C. Regularly change the bed linens
D. Performing insertion of catheter
UNIT EXAM 2 (MEDICAL-SURGICAL 1)

1. You are giving instruction to patient A on how to perform deep breathing and the use of incentive spirometry. The
patient asked you what the purpose of this was? The best response is?
A. Prevent COPD
B. Prevent Pneumonia
C. For peripheral circulation
D. Promote lung expansion

2.You are a nurse caring for a patient for scheduled BYPASS, you are giving the preoperative medication prior to
transporting the patient to the theater. What should you do after?
A. Placing the bed in a low position with side rails up (sir cabs)
B. Telling the patient he will be asleep for a long period of time
C. Encouraging patient to take a bath and change into OR gown after
D. Placing nasal cannula to the patient

3.You are caring for a patient with allergic rhinitis and ask you what should he do to decrease symptoms.Which of the
following is an appropriate action?
A. Encouraging to use nasal decongestant regularly
B. Encouraging to drink medication given
C. Instructing to increase the medication given
D. Instructing the patient to keep track of the occurrence with the symptoms

RATIO FOR A: Nasal decongestant sprays should not be used for more than two to three days at a time, because
they may cause a type of rhinitis called "rhinitis medicamentosa," which causes the nose to be congested constantly
unless the medication is used repeatedly.

4.You are caring for a patient recovering from an upper respiratory tract infection, what would be your expected outcome
of the recovery of your patient?
A. Maintaining Hydration status
B. Experiences less nasal obstruction
C. Experiences low grade fever on few several occasions
D. Cough with chest discomfort

5.Nursing responsibilities in INTRAOPERATIVE includes?


A. Maintain sterile technique while providing supplies and equipment for the sterile team
B. Coordinating patient care before, during and after surgical procedure
C. Recognizing and resolving environmental hazards that involve the patient or surgical team, including
protecting the patient from electrical hazards
D. Ensuring patient safety, positioning and monitoring and enforcing policies and procedures throughout
surgery- including a “time out”

RATIO FOR ALL: Tanan na include ani na site https://www.mometrix.com/academy/intraoperative-nursing/

6. All except one is TRUE about informed consent?


A. The nurse can witness the client signing the consent form
B. Patient under 18 years of age may need a parent or legal guardian to sign a consent form
C. It is the responsibility of the nurse to obtaining the consent for surgery
D. It is the nurse’s responsibility to ensure the patient has been educated by the physician about the procedure
before informed consent is obtained.

RATIO FOR C: Obtaining patients' informed consent is the physician's responsibility, but the process is more than just a
signature on a page. Surgery center staff are witnesses who confirm the informed consent form has been signed.
NURSES ARE ONLY THERE TO BE WITNESSES.

7. Postoperative patient is very much prone to infection, which of the following are indication of patient acquiring infection
A. WBC of 66,000mcl
B. Redness and Tenderness on the site
C. Presence of indwelling catheter for 2 days
D. Infiltrated IVF

8. You are a PACU nurse caring for a postoperative patient under General anesthesia. What is the potential risk of this
type of anesthesia?
A. Anemia
B. Pneumonia
C. Hypothermia
D. Atelectasis
9. You are caring for a patient semi-comatose post operatively, upon assessment all vital signs are within normal ranges.
What should be the best position for this type of patient?
A. Side lying
B. Prone
C. Fowlers
D. Semi-fowlers

RATIO FOR A: A patient who is semicomatose is at risk for aspiration (due to secretions pooling in the mouth or vomiting
which is a common side effect of sedation). Placing the patient onto their side preferably the left will help decrease the risk
of aspiration and help promote cardiovascular circulation.

10. Patient that is placed on NPO prior to surgery is very much important, the reason for this is?
A. Prevent development of diarrhea during giving of muscle relaxant
B. Prevents reabsorption of anesthetic agents given intra operatively
C. Preventing aspiration (sir cabs)
D. Preventing development of pneumonia post-operatively

RATIO FOR C: It is very important for every patient to have an empty stomach before any surgery or procedure that
requires anesthesia, for two reasons to keep any food or liquid from getting into the lungs.

11.You are caring for a postoperative patient under general anesthesia. The first assessment that you would make is?
A. Pain level
B. Breathing pattern (if select all that apply)
C. Airway patency
D. Perfusion
E. Heart rate

RATIO FOR C: Initial patient assessment should include: physical assessment of patient including airway, breathing,
circulation & disability (If select all that apply gani ni, include B and D)

12. Nurse K is caring for Patient I, a 70 year old female scheduled for OOPHORECTOMY. You know that most of the
eldely are more anxious compared to younger or middle aged. To alleviate anxiety you would?
Allow to rest
A. Administer sedative PRN IVTT route
B. Therapeutic Touch
C. Give PO morphine and monitor for RR distress
D. Palpate the bladder for any distention

13. You are admitting a patient with an insulin dependent to the unit for surgery. How should this patient diagnose of DM
type 1 affects their overall care?
A. You must assess the HGT of the patient (sir cabs)
B. You must give bolus IV dextrose as soon as possible
C. You should give the remaining insulin prior to surgery
D. You must place the patient on NPO for 12 hours
RATIO FOR A: It is recommended to check the blood glucose in the preoperative area.

14. You are caring for a postoperative patient following abdominal surgery. Patient stated “I don’t want to use my pain
medications because they will make on dependent” Which response by you is important?
A. “You need medication at least just a week”
B. “Pain medication will help you feel better, increase ability to breathe easier and the recovery is much fast
C. “Pain medication taken by mouth cannot make you dependent on the,”
D. “Taking the pain medication will help you move faster and also for faster healing” (sir cabs)

15. You are performing a preoperative assessment for patient undergoing Cataract surgery. While conducting the medical
history, you note that the patient has a kidney transplant about 5 years ago and is taking the prescribed
immunosuppressive drugs. Based on this data patient is at greater risk for
A. Infection
B. Hyperglycemia
C. Inflammation
D. Hypotension

RATIO FOR A: However, all immunosuppressant drugs carry the serious risk of infection. When an immunosuppressant
drug weakens your immune system, your body becomes less resistant to infection. That means they make you more likely
to get infections. It also means that any infections get will be harder to treat.
16. Patient told you that she is very nervous and really doesn’t understand the procedure appendectomy that was told by
her physician and surgeon. Is the appropriate action for this is?
A. Provide a patient with a graphical presentation and information about appendectomy?
B. Allow patient to relax; give PRN sedatives
C. Explain the procedure again in clear and calm manner together with the family
D. Call the physician and surgeon so that it can be explain again

RATIO FOR D: The nurse clarifies the information provided, and if the patient requests additional information, the nurse
notifies the physician. The nurse can only explain the nursing care that will take place after the procedure or treatment,
what medications you administered or will administer, and any other aspect of nursing care.

17. You are formulating a plan of care for patient. Astra a 60 year old patient who just had prostate operation. Which of the
following actions is necessary to prevent pneumonia?
A. Reposition every 2 hours
B. Encourage to increase oral intake to maximum level
C. Use egg crate mattress
D. Encourage to used incentive spirometry for lung expansion

RATIO FOR D: An incentive spirometer can keep the lungs active during bed rest. Keeping the lungs active with a
spirometer is thought to lower the risk of developing complications like atelectasis, pneumonia, bronchospasms, and
respiratory failure.

18. You are caring for a patient with moderate PNEUMONIA. What will be your appropriate nursing diagnosis?
A. Pain
B. Fluid imbalance (fluids and electrolyte ni siya and dili ni priority nga diagnosis)
C. Infection
D. Airway Clearance

RATIO FOR D: Ineffective Airway Clearance is a common NANDA-I nursing diagnosis for pneumonia nursing care plans.
This diagnosis is related to excessive secretions and ineffective cough or nonproductive coughing. Inflammation and
increased secretions in pneumonia make it difficult to maintain a patent airway.

19. What are the responsibilities of the PACU nurse?


A. Determining the incision site and assessing for infections immediately
B. Determining the anesthesia will subside and endorsing it to the floor nurse
C. Determining the name of drug and dosage used during surgery (not post-anes kay during surgery, pero pwede
sad)
D. Determining the significant others or family member of the patient (pre-anes, walay apil)
E. Determining pre operative level of consciousness (pre-operative meaning wala pa gi operate, walay apil for me)
F. Determining alleviation of pain and giving PRN drugs

20. All except one is false about ARDS?


A. Pneumonectomy can cure ARDS
B. All patient with ARDS will require extra oxygen
C. Acute respiratory distress syndrome is a life-threatening lung injury that allows fluid to leak into the lungs
(UNKNOWN)
D. ARDS patient may be given diuretic treatment

RATIO FOR B: All patients with ARDS will require extra oxygen. Oxygen alone is usually not enough, and high levels of
oxygen can also injure the lung. A ventilator is a machine used to open airspaces that have shut down and help with the
work of breathing.

21. All except one are manifestations of development of PTB?


A. Productive cough
B. Hemoptysis
C. Chills
D. Diaphoresis
E. Low grade fever
F. High grade fever

22. A student nurse was asked by one of the hospital staff the importance of PACU. Is the student nurse correct if he
states that?
A. PACU allows post-operative patient to recover from the effects of anesthesia, and the patient usually
stays in PACU until the patient is oriented, stable vital signs, and is without complications
B. PACU is an excellent place to triage patient
C. PACU helps patient relax and calm unlike in the medical surgical
D. PACU allows post-operative recovery from anesthesia in a stimulating environment to facilitate awakening and
orientation.
23. You are caring for a patient following a surgery. Patient is unable to move accordingly and is refusing to wear the
compression stocking because its discomfort to used. Refusing to wear the stocking places patient at risk for?
A. Pneumonia
B. Embolism
C. Varicose vein
D. Hematoma

24. You are caring a male patient scheduled for surgery. He stated “ I am really afraid , what if the surgeon will make a
huge mistake during surgery? Or what if I die during the operation!” What would be your best response?
A. “I will pray for you”
B. “Let’s talk about what you are feeling right now”
C. “You will be fine as what the doctor told you awhile ago”
D. “Don't worry you have the best surgical team”

25. You are discussing PTB with a group of people about PTB, one of the community members ask you what is the first
sign of PTB. You would correctly respond that the first and earliest sign is?
A. Cough with mucus (sir cabs)
B. Night sweat (late)
C. Hemoptysis (late sign) (if untreated)
D. Weight loss (late)

RATIO FOR A: The signs of active TB includes coughing for three or more weeks, coughing up blood or mucus, chest
pain, or pain with breathing or coughing.

26. You are providing follow-up check up to a patient with TB placed on short course chemo drug but according to
patient he does not regularly take his medication. Which action would be most appropriate
A. Ask the patient’s significant others to supervise the daily administration of the medications
B. Remind the patient that TB can be dangerous if not taken properly (pwede sab ni? Notify the physician
C. Visit clinic weekly to ask him whether he is taking his medications regularly

27. INH treatment causes peripheral neuropathies. Which of the following interventions would help prevent this
complication?
A. Giving patient with Thiamine
B. Giving patient with Riboflavin
C. Giving patient with Cyanocobalamin
D. Giving the patient with Pyridoxine

RATIO FOR D: Pyridoxine has been used to prevent or treat a certain nerve disorder (peripheral neuropathy) caused by
certain medications (such as isoniazid).

28. You know that a patient who is cyanotic that has pneumonia is primarily caused by?
A. Decreased O2 distribution
B. Decreased cardiac output
C. Increased hematocrit concentration
D. Increased WBC

RATIO FOR A: It is evident that the cyanosis of pneumonia patients is due to the incomplete saturation of venous blood
with oxygen in the lungs.

29. You are caring for a patient with pharyngitis and develops sore throat, your best action would be?
A. Offer warm saline for gargling
B. Offer a hot tea to promotes soothes
C. Offer cold water
D. Offer candy

RATIO FOR A: pharyngitis goes away on its own with salt water gargles, pain relievers and extra fluids to help alleviate
the symptoms.

30. You are caring for a patient who develops Atelectasis, you will likely to assess it.
A. Reddish sputum
B. Decreased temperature
C. Pain upon breathing
D. Severe cough

RATIO FOR C: To diagnose atelectasis, doctors usually start with X-rays (a test that provides pictures of the inside of
your chest). Another test called a computed tomography (CT) scan can provide more detailed pictures. In more severe
cases, a doctor may use a procedure called a bronchoscopy to see inside your airway.
31. You are caring for a postoperative patient to prevent immobility, you must?
A. Ambulate patient
B. Administer pain medication
C. Assist in ROM
D. Turn every 2 hours

32. ​You are caring for a post operative patient following abdominal surgery. Patient stated “ I don’t want to use my pain
medications because they will make on dependent.” Which response by you is important?
A. “You need medication atleast just a week.”
B. “Pain medication will help you feel better, increase ability to breathe easier and the recovery is much faster.”
C. “Pain medication taken by mouth cannot make you dependent on them.”
D. “Taking the pain medication will help you move faster and also for faster healing.” (sir cabs)

33. A type of surgical procedure that aims to restore function to traumatized or malfunctioning tissue and to improve self
concept.
A. Diagnostic
B. Re-constructive
C. Palliative
D. Curative

RATIO B: Reconstructive surgery is a procedure that restores your body after an injury, after a disease, or it corrects
defects you were born with. It not only restores your body, but your comfort and confidence.

34. You are a PACU nurse monitoring 4 patients. Which of the following manifestation from them requires notifying the
physician?
A. Patient 20 with BP of 100/75 mmHg (normal)
B. Patient 33 with temperature of 36 degree Celsius
- Hypothermia is a common complication of anesthesia
C. Patient 55 with total urine output for 24 hours of 300ml
D. Patient 17 with pain scale of 4 as 10 is the highest

35. You are caring for a 77-year-old female scheduled for surgery for treatment of CANCER. What appropriate diagnosis
should you prioritize?
A. Urinary Incontinence
B. Risk for Infection (sir Cabs)
C. Impaired Skin integrity
D. Airway Clearance

36. Patient Sinovac upon recovering from a surgery becomes restless , the pulse rate is 135 beats/min and blood
pressure is 80/50 mmHg, patient is cold, pale and clammy. The best action for this is?
A. Continue to monitor the status of patient
B. Monitor the blood glucose test and watch out for any signs of Hypoglycemia
C. Notify the Physician (sir cabs)
D. Get the ECG according

37. Upon the pre assessment phase you conducted a n interview for a patient that will soon be scheduled for Abdominal
Surgery. What important questions must you obtain from the patient in terms of safety?
A. How long do you expect to be discharged after the surgery?
B. Do you have any allergy to seafoods?
C. What types of food do you usually eat?
D. What prescription and nonprescription medication do you take? (sir cabs)

38. You are caring for a female patient for chest x-ray to rule out lung consolidation. What important assessment must be
obtain?
A. Pregnancy
B. Allergies
C. Kidney Problem
D. Claustrophobic

39. You are caring for a patient being discharged from the OPD following general anesthesia. What important instruction
you should give to your patient?
A. Encourage the patient to be admitted for close monitoring of the condition
B. Instruct the patient to increase oral intake immediately
C. Instruct the patient never to drive (kani)
D. Encourage the patient to eat protein products
40. Patient that is placed on NPO prior to surgery is very much important, the reason for this is?
A. Preventing development of pneumonia post- operatively
B. Prevent development of diarrhea during giving of muscle relaxant
C. Prevents reabsorption of anesthetic agents given intra operatively
D. Preventing of aspiration

41. You are a nurse on the operating room performing pre- operative assessment on patient subject for surgery. Patient
informs you that he drinks approximately 2 bottles of red wine each day and has been drinking for the last several years.
What post operative problems can you anticipate for this patient?
A. Alcohol withdrawal syndrome immediately following surgery
B. Alcohol withdrawal syndrome 2 to 4 days after his last alcohol drink (sir cabs)
C. Alcohol withdrawal syndrome 1 week after his last Alcohol drink
D. Alcohol withdrawal syndrome upon administration of general anesthesia

42. You are caring for a patient with flu and upon assessment the patient is experiencing fever with chills. Patient also
has productive cough with yellowish sputum. Upon auscultation crackles sounds is appreciated. You would interpret
this finding as?
A. Develops pneumonia
B. Normal in patient with Influenza
C. Acquired Bronchospasm
D. Develops atelectasis

RATIO FOR A: Pneumonia is an infection that inflames your lungs' bronchioles and air sacs (alveoli). The air sacs may fill
up with fluid or pus, causing symptoms such as a cough, fever, chills and trouble breathing. With pneumonia, you may
cough up phlegm that is yellow, green, or sometimes bloody.

43. Patient Ann is admitted to the emergency department in Wuhan hospital complaining of pain in the abdomen and
stated to you that “I vomited twice and it looks like coffee”. After evaluation patient Ann is diagnosed with a perforated
gastric ulcer and informed by the surgeon that patient Ann needs to undergo surgery. Patient Ann can anticipate the
surgery on?
A. As soon as possible (sir cabs)
B. Within 24 hours after her admission
C. Within next week after all diagnostic exams and procedures
D. Depends on the availability of the patient

RATIO FOR A: Despite arguments favoring conservative management of patients with a perforated peptic ulcer, delaying
the initiation of surgery more than 12 hours after presentation is associated with poor outcomes. Therefore, when
indicated, a laparotomy should be performed as soon as possible.

44. ​Nursing Responsibilities in the INTRA-OPERATIVE INCLUDES?


Select the correct response(s): [ALL ANG ANSWER ANI]
A. Ensuring patient safety, positioning and monitoring the patient, and enforcing policies and procedures
throughout the surgery – including a “time out”
B. Recognizing and resolving environmental hazards that involve the patient or surgical team, including
protecting the patient from electrical hazards
C. Coordinating patient care before, during, and after the surgical procedure
D. Maintaining sterile technique while providing supplies and equipment for the sterile team

45. ​When performing pre admission assessment to a patient for Orthopedic surgery. You know the main purpose of pre
admission assessment is to ?
A. Reviewing the health care insurance of the patient
B. Gauging the understanding of the patient as to why patient is admitted
C. Determining the needs for transportation post operatively
D. Verifies completion of pre operative diagnostic testing (pre-ad = pre-op) (sir cabs)
E. Determining possible complications post operative

46. You are assisting the patient for surgery pre operatively. What pre-operative assessments/ interventions should be
accomplished at this phase?
Select the correct response(s):
A. Establishing IV line to the patient (sir cabs)
B. Applying ground devices (intra ni)
C. Giving the patient anesthetic (intra nasad ni)
D. Vital signs taking and necessary information’s (sir cabs)
E. Verifying the surgical site (sir cabs)
47. You are performing a pre operative assessment of an elderly patient for thoracic surgery . The primary consideration
for elderly is very much important because?
A. Elderly requires higher medications
B. Elderly has smaller lung capacity
C. Elderly has less physiologic reserve (sir cabs)
D. Elderly has sophisticated coping skills

RATIO FOR C: Older adult patients have less physiologic reserve (i.e., the ability of an organ to return to normal after a
disturbance in its equilibrium) than younger patients

48. Which of the following can precipitate to EPISTAXIS ?


Select the correct response(s):
A. Trauma
B. Nose malignancy
C. English speaking People
D. Noise Blowing
E. Surgery of the nose

49. You are caring for a patient post 5 hours post-op with FBC attached to UB with level of less than a hundred. The Foley
bag catheter is patent upon assessment. Your next action would be?
A. Give Bolus of present IVF
B. Palpate for distended bladder
C. Irrigate the FB catheter with 50 ml saline in the port
D. Notify the physician

50. You are a PACU nurse assessing a patient post operative from surgery, you notice that the wound drainage
increased, patient is somewhat restless and upon taking the blood pressure it is declining and becomes unappreciated,
also an escalation of pulse noted. These finding are indicative of?
A. Thrombophlebitis
B. Atelectasis
C. Pneumonia
D. Hemorrhage
- When blood loss is 15 to 40%, mean arterial and pulse pressures fall, and heart rate increases, with the
magnitude of these changes being related to how much blood is lost. “WOUND DRAINAGE INCREASED”

51. To determine what type of antibiotics to be given to a patient with upper infections the nurse must ?
A. Request an order of MRI
B. Request an order of mid stream catch urine
C. Request an order of C/S (kani)
D. Request an order of sputum exam

52. You are caring for a 3 days post operative patient and reports to you of left calf pain, further assessment reveals of
redness and swelling. These is an indicative sign of complication of?
A. Atelectasis
B. Wound Infection
C. Deep Vein Thrombosis
D. Dysuria

53. You are caring for an elderly patient recovering from pneumonia. What expected assessment must be determined?
A. Maximum loss of 5 to 10 pounds of body weight.
B. Respiratory rate of 25 to 30 breaths per minute. (mali ni kay hyperventilation ni)
C. Chest pain that is minimized by splinting the ribcage.
D. Ability to perform ADLs without shortness of breath

54. You are teaching a patient recently diagnosed with PTB on how to prevent spreading the disease to the members of
the family . Which statement indicates proper understanding?
Select the correct response(s):
A. “I should use paper tissues to cough in and dispose of them properly.”
B. “It is important that I isolate myself from family when possible.”
C. “I should always cover my mouth and nose when sneezing.”
D. “I can use regular plates and utensils whenever I eat.”
E. “I will need to dispose of my old clothing when I return home.”

55. You are caring for patient post-operative of Colon Resection. Upon Positioning patient to the left would dehiscence
with evisceration occurs. What would be your action?
A. Splint the area with sterile OS (splinting is sa oxygenation ni siya nga concept)
B. Place a dressing saline soaked on the wound
C. Notify the physician immediately
D. Apply abdominal Binder
56. You are caring for a patient scheduled for surgery. Which of the following laboratory result of the patient may indicate
to notify the surgeon immodestly
A. hgt og 5.0mmo/L
B. platelet of 400,000/cu.mm
C. hct of 50%
D. hgb of .5 mg/dl

57. You are caring for a patient with ARDS, the earliest sign if ARDS is?
A. Increased Respiratory Rate
B. Crackles
C. Hypoventilation
D. Wheezing

RATIO FOR A: The first symptom of ARDS is usually shortness of breath. Other signs and symptoms of ARDS are low
blood oxygen, rapid breathing, and clicking, bubbling, or rattling sounds in the lungs when breathing

58. The most reliable assessment for air index to a patient is?
A. Listening and Feeling for the air movement (naa sa video daw)
B. Determining the sputum
C. Observing the skin color
D. Determining the rise and fall of the chest

59. You are a PACU nurse receiving a patient post operative from Oesophagectomy. Which are important and necessary
assessment you need to determine?
Select the correct response(s):
A. Pupillary response (sir cabs)
B. Bowel function
C. Urinary output (Sir cabs)
D. Level of Comfort
E. Pain (sir cabs)

60. You are performing a Mantoux test for a patient with HIV. You note an 8 mm area of induration at the skin test site, you
would interpret this as an?
A. Negative
B. Repeat Test
C. Inconclusive
D. Positive

RATIO FOR D: Reactions between 5 to 15 mm in size can be considered positive depending on risk factors, health, and
medical history. All reactions over 15 mm are considered positive regardless of risk factors.

61. You are caring for a patient scheduled for surgery this morning. Pre operative orders are given . What important
aspect must you check?
A. Verify if all laboratory results are normal range (dili sad tanan lab results normal pag surgery)
B. Have all consent signed
C. Removing of valuable jewelries
D. Ensure that patient changes into OR gown

RATIO FOR B: Kay dili maka proceed sa operation if wala naka sign sa informed consent??? TRUE MELS

62. You are caring for patient with PTB what should be included in the dietary intake of the patient?
Select the correct response(s):
A. Grain
B. Meat (sir cabs
C. Broccoli
D. Spinach
E. Milk
F. Egg
G. Citrus Foods (sir cabs)
- Another reason to eat more citrus fruits. Vitamin C boosts efficacy of TB drugs. US researchers found that
intake of vitamin c along with tuberculosis drugs may help the patients recover faster.

63. You are caring for a patient scheduled for laparoscopic procedure, the patient is taking several medications as part of
his maintenance regimen. Which drug might cause post operative risk for patient?
A. Valium
B. Bisacodyl
C. Milk of Magnesia
D. Coumadin (Tom)
- Discontinue warfarin (coumadin) at least 5 days prior to surgery. Therefore, give last dose on day 6 to achieve 5
warfarin-free days if day of surgery = day 0. Unless target INR is 3.0 (range 2.5 to 3.5) stop 6 days prior.

64. You are caring for a patient to be wheeled in the operating room for GI surgery. The physician already made his pre
medication orders and is to be given prior to the surgery. Which pre-operative assessment medications must be given
accordingly to the patient?
Select the correct response:
A. Fentanyl citrate (sir cabs)
B. Diazepam (given after)
C. Morphine (can cause respiratory distress)
D. Ranitidine (given after)
E. Atropine
F. Cimetidine

65. You are caring for patient with pneumonia and upon assessment the temperature of patient is 39 degrees Celsius.
What is your priority intervention?
A. Position the patient every 2 hours
B. Regularly change the bed linens (sir cabs)
C. Perform insertion of catheter
D. Suctioning Airway Clearance

66. You are caring for a patient with severe pneumonia, the temperature upon assessment is 101° and 102°F and also
periods of diaphoresis. What is your best nursing action?
A. Increase the oral intake of the patient
B. Maintain a complete bed rest
C. Notify the physician
D. Place patient oxygen on 2-3LPM

67. To prevent atelectasis to a patient with fracture rib you must?


A. Encourage deep breathing exercises and coughing exercises
- Performing deep-breathing exercises (incentive spirometry) and using a device to assist with deep
coughing may help remove secretions and increase lung volume.
- Gentle exercise for broken ribs can also help to clear your mind and prevent depression or anxiety from
developing. Breathing exercises are the main type of rehab, but chest-stretching exercises for broken ribs
are effective as well.
- Inspiration (simple breathing) exercises expand the chest wall, exercise the inspiratory muscles, and
prevent atelectasis.
- Deep breathing exercises and coughing after surgery can reduce your risk of developing
atelectasis. If you smoke, you can lower your risk of developing the condition by quitting smoking before
any operation. (MAO NANI GUYSES)
B. Apply thoracic binder for support (pwede sab ni pero di ko sure)
- While thoracic binder do control pain, they have been linked to cause hypoventilation, atelectasis and
pneumonia. As a result, their use is not recommended.
C. Defer pain medication
D. Position patient in semi fowlers (basin maka samot sa fracture jk yes kyang dapat hiyang siya diba kay ma igo
bayas foam ang ribs)

68. The use of Heimlich maneuver is when patient is choking. Which volume of air is the basis for the efficacy of this
maneuver to expel foreign object in the larynx?
A. Residual Volume
B. Vital Capacity
C. Tidal Volume
- Each application of the maneuver caused 120 cc of air to be ejected, the animal's normal tidal volume.
This air exchange suggests that the maneuver may be an effective means of artificial respiration, and
studies are being carried out to evaluate this possibility. HM = Heimlich maneuver; E = early; L = late

69. You are caring for an older patient for surgery. As the patient is being prepped for the said procedure. It becomes
clearer to you that the patient doesn’t fully understand the risk and benefits that comes along with the procedure? Which
of the following must be obtain by you?
Select the correct response(s):
A. Witnessing the patient’s signature on the consent form.
B. Advocating for the patient by ensuring she is making an informed decision
C. Describing any alternatives to the procedure
D. Explaining the risks and benefits associated with the procedure.
[TRUE ALL]
70. You are caring for patient scheduled for reduction of size in the spinal tumor to relive pain. This type of surgical
classification is:
A. Laparoscopic
B. Curative
C. Surgery to remove all malignant (cancerous) tissue, which is meant to cure the disease. This includes removing
part or all of the cancerous organ or tissue and a small amount of healthy tissue around it.
D. Palliative
E. Diagnostic

71. You are an operating room nurse and will be assisting patient for a major surgery. What actions will you prioritize in
this phase?
A. Emotional Support (unsaon pag provide support nga operating room nurse man, ngekngok)
B. Physiologic Monitoring
C. Preventing Infections
D. Cognitive Status

72. You are caring for a patient who has low pain threshold and experiencing mild anxiety attack following abdominal
surgery. Which interventions helps decreases the level of anxiety and pain?
Select the correct response(s):
A. Allowing patient to increase activity level
B. Coughing and Breathing Exercises (sir cabs)
C. Administering opioid medication
- Opioid drugs work by mimicking natural opiates and linking to the same receptors in the brain, meaning
that their immediate effect is to reduce anxiety in the same way that endorphins would.
D. Placing patient on oxygen at 2 LPM via nasal cannula
E. Use a guide imagery along with PRN pain medication (sir cabs)
F. Notifying the doctor

73. As a nursing student the importance of peri-operative nursing is?


A. To repair or remove traumatized tissue and structures
B. To cure an illness or disease by removing the diseased tissue or organs
C. To cure patient holistically
D. To prevent progression of disease
E. To obtain tissue for examination
[TRUE ALL KAY PERIOPERATIVE MAN]

74. A patient was successfully discharge from the unit following a BYPASS SURGERY. Post operatively the nurse must
instruct the patient for?
A. When to return after fully recovered
B. When to return for follow up check-up
C. When to return for another surgery
D. When to seek medical attentions if any unusuality’s noted

75. You are teaching the patient on how to perform proper deep breathing and coughing exercise after surgery. What
actions should be your teaching on this matter?
A. Patient should take 3 deep breaths and exhale forcefully and take a quick short period cough deep within the
lungs
B. Patient should take a deep breath into the mouth, exhale through the mouth to minimize the pressure and
take a short breath and then proceed to cough from deep in the lungs (sir cabs)
C. The patient should use the incentive spirometry and perform this maneuver at least twice a day
D. Patient post operatively should take at least 1 deep breathing exercise and cough smoothly three times at least
every 20 minutes

76. The best time to instruct the patient about splinting the incision and leg exercises is?
A. After anesthesia will subside
B. At the post anesthetic care unit
C. Upon admission to the patient (sir cabs)
D. Upon wheeling the patient to the operating room

77. You are caring an assessing for a patient post-operatively that the left calf upon assessment is hard, tender and pain
is present as claimed by patient. What should be the appropriate action to this?
A. Instruct the patient not to sit for a long period of time
B. Instruct the patient not to dangle the legs or massage the affected area
C. Administer anticoagulant as ordered
D. Instruct the patient to elevate the extremities without allowing any pressure

78. You are obtaining the informed consent for a 16 year old who has just been married and expecting her first son.
Patient is for C-section. According also to the patient she is still living under her parents. You know that the legal person to
signed to consent is the?
A. Parents (not sure)
B. Husband (not sure)
- Basta married and right kay dili sa na parents kundi sa husband
C. Patient herself
D. Physician

79. You are a nurse preparing patient to be wheeled from the presurgical area in theater 1 and reviewing the informed
consent of the said patient. What are the criteria for legally valid informed consent?
A. Consent must be obtained by a physician (sir cabs)
B. Consent must be notarized (no need magpanotarize sa lawyer)
C. Consent must be signed on the day or surgery (consent is obtained after explanation sa surgery)
D. Signature must be witnessed by a professional staff member (sir cabs)
E. Consent must be freely given (sir cabs)
-consent must be freely and voluntarily given

80. Patient is pre medicated with BENZODIAZEPINES, the appropriate action by the nurse is?
A. Raising the side rails of the bed (sir cabs)
- Benzodiazepines cause acute adverse effects: drowsiness, increased reaction time, ataxia, motor
incoordination, and anterograde amnesia
B. Instruct the significant others to watch of for signs of fever and report them to the nurse on duty
C. Use infusion set upon giving the medication to the patient
D. Assist patient to the comfort room

81. You are caring for a patient with pneumonia and upon assessment the temperature of the patient is 39 degrees
celsius. What is your priority prevention?
A. Regularly change the bed linens (sir kabs)
B. Perform insertion of catheter
C. Position the patient every 2 hours
D. Suctioning airway clearance.

82. What of the following are manifestations of PHARYNGITIS?


A. Sore throat
B. Cough
- The main symptom of pharyngitis is a sore, dry, or itchy throat. Additional symptoms may appear
depending on the type of infection, such as cold or flu symptoms. Symptoms of viral pharyngitis include :
a cough. (same ratio)
C. Tachycardia
D. Fever

83. You are caring for patient with severe pneumonia. Which of the following manifestations would indicate changes in
sensorium of the patient?
A. Irritability
B. Coma
- A change in cognitive status (delirium, confusion, dementia) is a major sign of pneumonia in the elderly.
C. Apathy
D. Fever

84. You are caring for a patient with pulmonary hypertension, the doctor ordered diagnostic imagery to determine and
reveal filling defects in the pulmonary vasculature. This test is?
A. CT scan
B. ECG
C. Echocardiography
D. Pulmonary angiography

RATIO FOR D: Computed tomography pulmonary angiography (CTPA) has become the primary method used for the
diagnosis of PTE in patients, largely replacing the previous method of choice, pulmonary angiography.

85. You are caring for patient whose histories include different health related problems, these includes scarlet fever, otitis
media, bacterial endocarditis, rheumatic fever and glomerulonephritis. What common factors associated these diseases
should consider?
A. Caused by parasitic bacteria
B. Can be controlled by childhood vaccinations
C. Self limiting infections
D. Result in streptococcal infections enter via upper respiratory tract

86. You are caring for a newly admitted patient upon assessment you note that patient is bluish, what would be your
appropriate action?
A. Instruct to perform deep breathing exercise
B. Notify the physician
C. Give oxygen at 2LPM
D. Encourage to increase oral intake

87. The most common cause of bacteria in pharyngitis?


A. Streptococcus
-However, when the cause is bacterial, pharyngitis is most often caused by group A beta-hemolytic Streptococcus
(GABHS), commonly referred to as strep throat.
B. Streptococcus bovis
C. Group C streptococci, S
D. Streptococcus agalactiae

88. You are caring for a patient admitted to the respiratory unit with a diagnosis of pneumonia. Which of the following
actions must be performed?
Select the correct response(s):
A. Administering IVTT antibiotics as prescribed (sir cabs)
B. Checking the peak level of antibiotics (it is the doctor)
C. Intravenous insertion (sir cabs)
D. Obtaining health history (sir cabs)
E. Collecting sputum specimen (sir cabs)
F. Performing physical examination (sir cabs)

89. A type of anesthesia that uses local anesthetics to cause a temporary loss of….
A. General anesthesia
B. Epidural anesthesia
C. Regional anesthesia
- Local anesthetics cause a temporary blocking of conduction along these nerve fibres, producing a temporary
loss of pain sensation.
D. Spinal anesthesia

90. The main purpose of letting patient perform leg exercise post operatively is to:
A. Leg exercises promotes venous blood return
B. Leg exercises prevent venous thrombosis
C. Leg exercises prevent development of pressure ulcer
D. Leg exercises increases muscle mass

RATIO FOR B: Simple exercises while you are resting in bed or sitting in a chair can help prevent blood clots. Move your
feet in a circle or up and down. Do this 10 times an hour to improve circulation. Ambulation (getting out of bed and
walking).

91. All except one are included in the responsibility of the nurses prior to admitting…. [INCOMPLETE]
A. Verifies completion of preoperative testing
B. Assess for risk for postoperative complications
C. Verifies completion of preoperative testing
D. Reinforce previous teaching
E. Wala diri ang answer, ambot asa ge butang ni sir

92. You are preparing the patient for surgery for HYSTERECTOMY without removing the ovaries as indicated. You are
witnessing….. Would be the best indicate the understanding of an informed consent? [INCOMPLETE]
A. “The surgeon will remove a part of my body and told me that there are some risk after like bleeding and
pain and I should….. (NAPUTOL)
B. “Because of my uterus is not function anymore it should be removed and we have to consult and think about
adopting”
C. “I know after the removal of some parts of my body I will feel pain, but you will help me right? In minimizing the
pain….. (NAPUTOL NASAB)
D. “After the surgery I know I will be fine because it was guaranteed by my surgeon”

93. You are caring for a patient who is scheduled for removal of tumor. The patient is anxious and apprehensive and
keeps asking you about anything and about medicine pre operatively. The medicine is to be given on call to OR. When
would be the best time to administer the medication to the patient?
A. As soon as possible to decrease the anxiety of patient
B. As soon after being notified by the OR staff
C. As soon as anxiety will subside
D. As soon as patient is being transferred to the OR table

94. You are preparing to transport patient to the OR for his scheduled surgery. What should you ensure on the chart when
endorsing it to the OR staff?
A. Vital Signs Monitoring)
B. Social Work Assessment
C. Verified Consent)
D. Laboratory Reports
E. Nurses Note
F. Dietician Assessment

95. You are caring for a patient just had his bronchoscopy. what important health teaching must be provided to the
patient?
A. Increase the IVF rate
B. Maintain NPO status until further orders by the physician
C. Assess the gag reflex regularly
D. Encouraging to increase oral intake

96. You are caring for a patient post-op for 2 days and reports to you that she hasn’t had any bowel movement since her
surgery 2 days ago. Upon assessment , you note the distention of the abdomen and no appreciated bowel sound in the 4
quadrants. What action must you perform?
A. Encourage ambulation
B. Perform fleet enema
C. Give bolus of IVF atleast 100 ml
D. Instruct to eat foods high in fibers

97. Patient Delta is for bowel resection tomorrow at 9 in the morning and patient was given an order of cleansing enema.
The purpose of cleaning enema is that it has a therapeutic effect on?
A. Prevent accumulation of gastric bypass
B. Prevent Gastric aspiration
C. Prevent contaminating the peritoneum
D. Facilitate absorption of the intra operative medication

98. Which obtained health history per-operatively puts the patient in greater risk for surgery
A. Use of Cocaine
B. Hemoglobin of 10 gm/dL
C. Current UTI
D. History of Ventricular Septal Defect

99. You are caring for patient with PTB which earliest symptoms are expected?
A. High Grade Fever
B. Mental Status changes
C. Weight Loss
D. Hemoptysis

100. Which of the following are manifestations of Pulmonary Hypertension:


A. Agitated
B. Restlessness
C. Distended Neck Vein
D. Reddish copious phlegm
E. Reduced carotid pulse

101. You are caring for a patient with lung problem upon auscultation it shows crackling in the right lobe Your nursing
action to this is?
Auscultation of a client’s lungs reveals crackles in the left posterior base. The nursing intervention is to:
A. Instruct patient to increase intake
B. Position patient in semi fowlers
C. Determine signs of pulmonary edema
D. Repeat the auscultation and instruct patient to take a deep breathe or cough

102. Patient Pfizer is scheduled for PFT. You explain to the patient that during the procedure one the instruction that will
be given by the respiratory therapist is that you will breathe normally. Because these measures the?
A. Vital Capacity
B. Expiratory Reserve
C. Tidal Volume
D. Inspiratory Reserve

103. You are caring for patient with pneumonia what actions are necessary?
A. Assessing for peripheral edema.
B. Assessing the color of nail beds. (sir cabs)
C. Auscultating the bowel sounds.
D. Assessing for chest pain. (sir cabs)
E. Auscultating the breath sounds. (sir cabs)

104. You are caring for a patient with Pulmonary Hypertension the diet of choice for this patient is?
A. High in Protein
B. Antioxidant
C. Low salt
D. Low Potassium
UNIT EXAM 3 (MEDICAL-SURGICAL 1)

1. A 21-year old patient with PID is in distress and tells the nurse that she is afraid she will be sterile and not have kids.
What is the nurse’s best response?
A. Ang impeksyon pwede maka cause ug abscess ug sepsis, kini and mas dapat nato bantayan
B. Daku and posibilidad nga di na makaanak pag magka PID. Gusto ba nimu hisgutan nato imung gibati
bahin ani? [SG] (Ratio)
C. Sige lang Ma’am, everything happens for a reason
D. Di man common na mabaog pag magka PID

2. You are assessing a patient with HPV infection, what symptoms should you expect to find?
A. Multiple gray warts in the perineal area that are merging [SG]
B. The vulva has a painless, indurated lesion
C. Perineal vesicles and ulcerations are painful
D. Vaginal discharge that is purulent

3. A client with Guillan-Barré syndrome is being cared for by a nurse. The client is having trouble expectorating
secretions. Which of the following…. [INCOMPLETE]
A. Provide O2 at 2L/mon
B. Suction secretions (Ratio)
C. Auscultate for rales or wheezing
D. Place the patient in tripod position

4. Which of the following STIs, if present at the time of delivery, will indicate the necessity for cesarean section?
A. Genital herpes [SG]
B. Syphilis
C. Gonorrhea
D. Chlamydia

6. Plasmapheresis treatments are arranged for a client who has been diagnosed with GBS. The objective of
plasmapheresis, as explained by a nurse… [INCOMPLETE]
A. Restore albumin levels in the blood
B. Restore fluid balance in the body
C. Infusion lipoproteins to restore the myelin sheath
D. Remove circulating antibodies from the bloodstream

7. A nurse is caring for a client with SLE. SLE is a chronic autoimmune disorder that causes cutaneous and systemic
symptoms. The nurse… [INCOMPLETE]
A. Collagen overproduction wreaks havoc on internal organ function
B. Complement activation is triggered by the formation of aberrant IgG that binds to cellular antigens
C. Autoantibodies destroy nucleic acids and other self-proteins [SG] (Ratio)
D. T suppressor cell activity is increased in the presence of B-cell hypoactivity, leading in immunodeficiency

8. After an appendectomy, a patient with a ruptured appendix returns from the operating room with a JP. What is the
drain’s function?
A. Reduce the amount of discomfort after surgery
B. To reduce the formation of scar tissue
C. To drain wound exudates (Ratio)
D. It serves as a point of entry for wound irrigation

9. A doctor prescribed an enema for a patient who is suspected of having appendicitis. The nurse must… [INCOMPLETE]
A. Request that the patient lie of his left side
B. Clarify about the prescription with the doctor (Ratio)
C. Clients should be informed about the procedure
D. Assemble all necessary equipment

10. Which treatment should be included in the immediate management of acute appendicitis?
A. Relieve pain
B. Prevent fluid volume deficit
C. Administer antibiotic therapy
D. Reduce anxiety

11. Rheumatoid arthritis is an autoimmune disorder that causes an attack against the synovium. During the physical
assessment… [INCOMPLETE]
A. Crepitus on joint movement
B. Swelling of the hand at the knuckles
C. Heberden’s nodes
D. Enlargement of liver
12. A nurse is caring for 43-year old client in the neuro ward after a diagnosis of acute MS exacerbation. Which of the
following symptoms will the nurse expect to find?
A. Paresthesias, visual abnormalities, and motor impairment (Ratio)
B. Respiratory failure
C. Excessive involuntary movements, hearing loss, and ataxia
D. Ascending weakness of the lower extremities

13. Laboratory findings that the nurse would expect to be present in the patient with RA include
A. Anti-citrullinated protein antibody
B. Low white blood cell (WBC) count
C. Polycythemia
D. Increased immunoglobulin G (IgG)

14. Which of the following should be included in a 22 year-old female patient’s postoperative treatment after an
appendectomy?
A. Administer sitz baths TID
B. Measuring abdominal girth q 2 hours
C. Observing for first bowel movement after surgery (Ratio)
D. Complete bed rest without toilet privileges

15. Which of the following characterizes herpes simplex virus infection (select all that apply)?
A. Painful vesicular sores that rupture and ulcertate are the most common sign of genital herpes
B. When genital herpes lesions are present, condoms should be used to prevent transmission
C. HSV-2 is a kind of herpes simplex virus that only causes vaginal sores
D. Sexual activity and stress can trigger recurrent symptomatic genital herpes
E. Acyclovir can be used to treat genital herpes

16. What should the nurse include in the teaching plan for the patient with SLE?
A. Ways to avoid exposure to sunlight (Ratio)
B. The use of nonpharmacologic pain interventions instead of analgesics
C. Increasing dietary protein and carbohydrate intake
D. The necessity of genetic counseling before planning a family

17. A client with PID is being cared for by the nurse. Which of the following should be included in the care plan?
A. Ambulate the patient often to encourage exudate drainage
B. Put patient in a semi-position Fowler’s while on bed rest (Ratio)
C. Vaginal irrigations q 4 hrs
D. Instruct the patient to use tampons for the vaginal discharge

18. A ward nurse is caring for a female patient with GBS was admitted to the hospital for 72 hours now. Which of the
following symptoms will need…. [INCOMPLETE]
A. Seizures
B. Desquamation of skin
C. +4 DTR
D. Ascending weakness

19. Your 18 year old patient diagnosed with appendicitis develops fever, and has vital signs of HR of 120 and BP 80/50.
Which of the following… [INCOMPLETE]
A. Intestinal obstruction
B. Deficient fluid volume
C. Peritonitis
D. Bowel ischemia

20. A nurse is caring for a client newly diagnosed with MS. Multiple sclerosis is a debilitating disorder. THe nurse explains
that MS is diagnosed by [INCOMPLETE]
A. MRI findings
B. The patient’s medical history and clinical symptoms (Ratio)
C. Cerebrospinal fluid analysis
D. CRP and ESR findings

21. The nurse suspects a peritonitis in a patient with colon tumor when which of the following manifestation is observed?
A. Excessive flatulence
B. Explosive diarrhea
C. Rigid abdominal wall upon palpation
D. Hyperactive bowel sounds upon auscultation
22. Which of the following STI patient is most likely to avoid seeking and adhering to treatment for their infection?
A. A patient with genital herpes
B. A patient with gonorrhea
C. A patient with HPV infection
D. A patient with syphilis

23. To be diagnosed with Systemic Lupus Erythematosus is a life-changing diagnosis. A nurse is caring for a 20-year old
woman newly diagnosed with SLE is anxious. Which of the following replies will best address the client’s concern?
A. Most persons with SLE have remissions and exacerbations on a regular basis with fast progression to irreversible
organ impairment
B. People with SLE have a shorter life expectancy, although the condition may be managed with long-term
corticosteroids
C. Because SLE seldom results in death, you should expect to live a near-normal life
D. It’s impossible to say because the disease’s severity and course are so varied

24. Guillain Barre syndrome is disorder that causes the body’s immune cells attacks the peripheral myelin sheath. Which
of the following interventions… [INCOMPLETE]
A. Preparing the patient for intubation and attaching the patient to mechanical ventilator (Ratio)
B. Encouraging active exercises of the muscles
C. Teaching the patient to exercise facial muscles by drinking via straw
D. Giving cefuroxime 750mg IVTT q six hours as prophylaxis

25. Which of the following complications should a nurse keep an eye on if a client has Guillain-Barré syndrome?
[INCOMPLETE]
A. Failure of the respiratory system
B. Thrombus emboli
C. Dysreflexia of the autonomic nervous system
D. Intercranial pressure increase

26. A woman who has had her pelvic inflammatory condition related to chlamydia and gonorrhea treated is having trouble
conceiving. The doctor informs the… [INCOMPLETE]
A. The infection caused damage to the cervix, which caused it to close, preventing sperm from accessing the uterus
B. Because of the uterine damage produced by the infection, when fertilization occurs, the fertilized egg does not
implant into the uterus.
C. The scarring caused by the infection in the fallopian tubes has resulted in the tubes being permanently
blocked (Ratio)
D. Bacause the virus destroyed the woman’s ovaries, ovulation no longer occurs

27. A patient with Guillain-Barré syndrome asks “Maglisod naman ko ug ginhawa, mamatay na ba ko ani?” in responding
to the patient, what should the nurse know.. [INCOMPLETE]
A. Whe nerve injury reaches the brain and meninges, death ensues
B. The majority of people with Guillain-Barre syndrome recover completely (Ratio)
C. Residual paralysis and sensory impairment are typically permanent if death can be avoided
D. Parents who require mechanical ventilation nearly usually die

28. Fourteen days after being treated for a chlamydia infection, a patient returns to the clinic complaining again of a
urethral discharge. Which of the following… [INCOMPLETE]
A. Gainom man kos Vibramycin kaduha sa isa ka adlaw for 2 weeks
B. Wa pa nako maistoryahan akong girlfirend bahin sa akoang impeksyon (Ratio)
C. Ga inom mis akong mga amigo gabii while on antibiotics pako
D. Ka isa pako nakighilawas sugod atong nahuman kog inom sa akong antibiotics.

29. A woman with an intense rheumatoid arthritis flare-up informs the nurse that she is too exhausted to bathe. What is
the nurse’s role in this patient’s care?
A. Provide bed bath for the patient
B. Tell the patient self-care activities are important
C. Inform the patient that she may skip bathing for now
D. Let the patient rest first then assist the patient to the bath (Ratio)

30. The nurse is evaluating a male patient with gonorrhea, which of the following is the most important assessment should
the nurse ask about?
A. The date of his last sexual activity
B. His recent sexual partners
C. A prior history of STIs
D. When the symptoms began

1. Which of the following treatment will prevent stimulation of gallbladder?


A. Administration of anticholinergics
B. NPO with NG Suction (Ratio)
C. Administration of Plasil
D. CholeBPHcystectomy

2. A nurse is discharging an acute pancreatitis patient. Which of the following statements require further instructions?
A. “Dapat e monitor nako akong blood sugar hantod na ma akong pancreas”
B. “Dapat di nako mukaon ug salty foods” (Ratio)
C. “Dapat akong obserbahan akong tae kung naa bay fats”
D. “Dapat di nako muinom ug alcoholic drinks”

3. Which of the following of the manifestations suggest resolution of acute nephritis syndrome?
A. Urine output of 300mL in 8 hours (Ratio)
B. Increased GFR (K)
C. Absence of blood in the urine
D. Absence of protein in the urine

4. What is the most common cause of secondary immunodeficiency disorders?


A. Chronic stress
B. T-cell deficiency from HIV
C. Drug-induced immunosuppression
D. Common variable hypogammaglobulinemia

5. Which of the following is NOT an expected diagnostic finding is Acute Nephritic Syndrome
A. Creatinine and BUA are increased
B. Elevated Ig A
C. Increased complement level
D. Anemia

6.A diabetic patient is learning how to combine a regular insulin and NPH insulin in one syringe. Which of the following if
done by the patient is incorrect?
A. Withdraws the NPH dose into the syringe first (Ratio)
B. Adds air equal to the insulin dose into the regular vial and withdraws the first insulin
C. Removes any air bubbles after withdrawing the first insulin
D. Injects air equal to the NPH dose into the NPH vial first

7. You are caring for BPH patient in the surgical ward. The patient asks what is the difference between BPH and prostate
cancer. The best… [INCOMPLETE]
A. BPH is a benign tumor that does not spread beyond the prostate gland (Ratio)
B. BPh is an enlargement of the gland caused by an increase in the size of existing cells
C. BPH is a precursor to prostate cancer but does not yet show any malignant changes
D. BPH is a benign enlargement of gland caused by an increase number of normal cells

(4 CORRECT ANSWERS)
8. Which of the following laboratory finding is expected in patient with acute pancreatitis?
A. Decreased serum glucose
B. Decreased urinary amylase
C. Increased serum amylase
D. Decrease serum calcium

9. Which of the following physical assessment procedure is included in the assessment for the presence of BPH?
A. Palpating the base of the penis for enlargement
B. Palpating the scrotum and testes for mass
C. A digital rectal examination to palpate the prostate gland (Ratio)
D. Palpating the inguinal ring while the patient bears down

10. What is meant by a type IV or delayed hypersensitivity reaction?


A. Following antigen-antibody interaction on cell surfaces, complement activation causes cellular lysis or
phagocytosis
B. Antigen interactions with particular IgE antibodies attached to mast cells or basophils, resulting in the release of
chemical….
C. T cells that have been sensitized or regenerate cytokines that attract macrophages that cause tissue
damage [SG]
D. Antigens couples with IgM that are too tiny to be eliminated by the mononuclear phagocytic system accumulate
in…

11. A 19 year old male patient with a diagnosis of Crohn's disease is admitted to with persistent diarrhea. What are
characteristic of Crohn’s disease (select all that apply)
A. Hematochezia
B. Has segmented distribution [SG]
C. Toxic megacolon
D. Abdominal pain [SG]
E. Involves the entire thickness of the bowel wall
F. Weight Loss [SG]

12.When a nurse examines a patient with severe pancreatitis, she expects to find
A. Hyperactive bowel sounds
B. Temperature > 39 degrees celsius
C. BP of 140/90, Hr of 110
D. Severe midepigastric or left lower quadrant (LUQ) pain

13. Cholecystectomy can be done through a laparotomy or through a laparoscopy. Which of the following interventions is
expected post laparoscopic cholecystectomy?
A. Be hospitalized for 3 to 5 days following a surgery
B. To facilitate bile drainage, a T-tube should be inserted in the common bile duct
C. Dressing of four small abdominal incisions (Ratio)
D. Return of four abdominal incisions

14.You are caring for a client with a renal stone passing down the ureter. Which of the following assessment findings are
expected of the patient?
A. Recurrent UTIs
B. Distended bladder region
C. Severe, colicky back pain radiating the groin [SG] (Ratio)
D. Hesitancy and urgency in urination

15.A recently hospitalized diabetic patient is being evaluated by the nurse. Which of the following observations should be
considered as a priority… [INCOMPLETE]
A. Bilateral numbness of both hands
B. Areas of lumps and dents on the abdomen
C. Rapid respirations with deep inspiration [SG] (Ratio)
D. Stage II pressure ulcer on the right heel

16. Patient who undergoes lithotripsy are at risk for infection due to introduction of bacteria in the urinary tract. Which of
the following intervention… [INCOMPLETE]
A. Collect urine for culture and sensitivity
B. Apply cold compress to the flank area
C. Watch out for bloody urine
D. Encourage fluid intake of 3L/day [SG]

17. What are manifestations of diabetic ketoacidosis (DKA) (select all that apply) (all choices)
A. Thirst [SG]
B. Sweet, fruity breath odor [SG]
C. Kussmaul respirations [SG]
D. Ketonuria [SG]
E. a acidosis [SG]
F. Dehydration [SG]

18. A patient with inflammatory bowel disease has a nursing diagnosis of imbalanced nutrition: less than body
requirements related to decreased intestinal absorption. Which assessment data supports this nursing diagnosis?
A. Anorectal excoriation and pain
B. Pallor and hair loss [SG] (Ratio)
C. Frequent diarrhea stools
D. Hypotension and urine output below 30 mL/hr

19. (walay pic)

20.Which of the following statements describe acute transplant rejections?


A. Is reversible with immunosuppressive treatment [SG]
B. Only transplanted kidneys are affected
C. Use of supportive management
D. When this happens, the organ must be removed
E. T cytotoxic in the recipient assault the foreign organ [SG]
F. Immunosuppressants must be used for a long time to prevent rejection [SG]

21. Why is plasmapheresis recommended for the treatment of autoimmune diseases?


A. Obtain plasma for the purpose of analyzing and evaluating certain autoantibodies
B. Increase the number of monocytes in the circulation to encourage the clearance of immunological complexes…
C. Remove autoantibodies, antigen-antibody complexes and immune-inflammatory mediators [SG]
D. Reduce the number of lymphocytes in the blood to inhibit immunological reactions

22. You are caring for a post-TURP client with continuous bladder irrigation. A couple of hours after the surgery, the
catheter… [INCOMPLETE]
A. Take the patient’s vital signs
B. Call the physician right away
C. Clamp the tube with a forcep
D. Increase the rate of the PNSS irrigation and take vital signs [SG] (Ratio)

23… four consecutive mornings, a patient on insulin has had fasting glucose readings above 200 mg/dl (11.1mmo/L) upon
awakening. What should the nu… [INCOMPLETE]
A. To avoid the dawn phenomenon, increase your insulin dose in the evening
B. Use an intermediate-acting insulin and a single-dose insulin regimen
C. Check the glucose level before going to bed, between 2:00 and 4:00 am an when patient wakes up [SG]
D. Reduce your evening insulin dose to avoid nocturnal hypoglycemia and the somogyl effect

24.Which of the following are cholelithiasis risk factors?


A. Age of 40 and above [SG] (Ratio)
B. Multiparity [SG] (Ratio)
C. Oral Contraceptives or estrogen use [SG] (Ratio)
D. Obesity [SG] (Ratio)
E. Excessive alcohol consumption in the past (way labot)
F. Gallbladder illness in the family [SG] (Ratio)

25. After receiving an IM injection of penicillin in the gluteus maximus, a patient suddenly suffers dyspnea and paralysis.
Which of the following add… reaction [INCOMPLETE]
A. Pulse rate of 55
B. Anxiety (APIL NI IF SELECT ALL THAT APPLY)
C. Pupil constriction
D. Wheezing [SG]
E. BP of 140/90

26. Acute pancreatitis is a rare autoimmune disorder that causes severe pain.In addition to effective use of analgesics.
What should the nurse include… [INCOMPLETE]
A. Shift the three meals into 6 smaller meals
B. Ambulate the patient every 3-4 hours
C. For the pain relief the patient on their side with the head of bed elevated 45 degrees [SG]
D. Provide entertaining activities to take his or her mind off the discomfort

27.One of the pharmacologic treatment for BPH is finasteride (Proscar) What is the effect of this drug?
A. It stimulates androgen production
B. It tones the bladder detrusor muscle promoting urine flow
C. It reduces the size of the prostate gland [SG]
D. It increase bladder tone

28.Which of the following is a primary acute nephritic syndrome


A. Abnormal concentration of nitrogenous wastes in the blood
B. Dehydration and thirst
C. Excess sugar in the blood
D. Headache and seizure
UNIT EXAM 4 (MEDICAL-SURGICAL 1)

1. Which of the following tests will the physician order if he wants to know if trastuzumab (Herceptin) will be effective for a
breast cancer patient?
a. BRCA1 gene mutation
b. HER2 protein levels [SG]
- Trastuzumab is effective only for women whose breast cancer tumors have genes that overproduce the
protein HER-2. Testing for BRCA genes is used to identify women at risk for developing breast cancer.
The stage of cancer is not relevant to whether this drug should be used.
c. Stage II cancer identification
d. BRCA2 gene mutation

2. A nurse is assessing a patient with colorectal cancer. Which of the following risk factors should the nurse ask about?
a. Dietary intake [SG] (Ratio)
- A diet high in red meat and low in fruit and vegetable intake is associated with development of CRC, as
are alcohol intake and smoking. Family and personal history of CRC also increases the risk. Other
environmental agents are not known to be related to CRC. Long-term use of nonsteroidal
antiinflammatory drugs (NSAIDs) is associated with reduced CRC risk.
b. Environmental exposure to carcinogens
c. Long-term use of nonsteroidal anti-inflammatory drugs (NSAIDs)
d. Sports involvement

3. What teaching the patient with cancer about chemotherapy, which approach should the nurse take?
a. Inform the patient that chemotherapy-related alopecia is usually permanent but can be managed with lifelong use
of wigs
b. Assure the patient that the side effects from chemotherapy are uncomfortable but never life threatening
c. Avoid telling the patient about possible side effects of the drugs to prevent anticipatory anxiety
d. Explain that antiemetics, antidiarrheals, and analgesics will be provided as needed to control side effects
[SG]
- Patients should always be taught what to expect during a course of chemotherapy, including side effects
and expected outcome. Side effects of chemotherapy are serious, but it is important that patients be
informed about what measures can be taken to help them cope with the side effects of therapy. Hair loss
related to chemotherapy is usually reversible and wigs, scarves, or turbans can be used during and
following chemotherapy until the hair grows back.

4. Which of the following statements is true regarding lung cancer types?


a. Small cell carcinoma is the aggressive type of lung cancer [TRANS]
b. Small cell carcinoma causes a peripheral mass in the bronchi
c. Adenocarcinoma present more in the central portion of the lungs
d. Non-small cell carcinoma causes early mediastinal involvement

5. A patient with gastric cancer is manifesting hematemesis, which of the following is the most appropriate nursing
diagnosis for the patient?
a. Fluid volume deficit (Ratio)
b. Impaired gas exchange
c. Imparied oral mucous membrane
d. Decreased cardiac output

6. The following types of breast cancer generally have favorable diagnosis:


a. Mucoid carcinoma [TRANS]
b. Medullary breast cancer [TRANS]
c. DCIS [TRANS]
d. Paget’s disease
e. Inflammatory breast cancer
f. Ductal carcinoma in situ [TRANS]

7. Which of the following symptoms of lung cancer may indicate spread to the mediastinum?
a. Sudden change in the quality of cough
b. Dull, aching chest pain [TRANS]
c. Pleuritic pain
d. Wheezing

8. Which of the following are cancer promoters?


a. HPV
b. Sedentary lifestyle
c. X-ray radiation
d. Unhealthy diet
e. Cigarette smoking
9. Which of the following are prophylactic measures against ovarian cancer
A. Safe sexual practices
B. Delaying sexual debut
C. Bilateral mastectomy
D. Use of oral contraceptives
E. Salpingectomy and oophorectomy

10. What is the treatment of choice of small-cell carcinoma?


A. Chemotherapy
B. Pneumonectomy
C. Lobectomy
D. SBRT

11. Which of the following is the primary use of biological therapy in cancer?
A. Prevent fatigue caused by chemotherapy and high-dose radiation which is evident in bone marrow depression
B. Reduce the immune system and circulating lymphocytes while increasing feelings of well-being by compensating
for central nervous system inadequacies
C. During chemotherapy, protect normal, quickly reproducing gastrointestinal cells from being harmed
D. Improve or complement the host’s immunological responses against tumor cells that cause flu-like
symptoms [SG]
- Immunotherapy uses normal components of the immune system and is used therapeutically to boost or
manipulate the immune system to create an environment not conducive for cancer cell growth or that
attacks cancer cells directly. Immunotherapies may cause flu-like symptoms. The other options are not
correct.

12. Which of the following are risk factors for breast cancer?
A. Women whose sisters or mothers have had breast cancer are at increased risk
B. Nulliparous women are at increased risk
C. Breast cancer occurs most frequently in women younger than 30 years
D. Risk is increased in postmenopausal women with body mass indexes below 20
E. The longer the interval between menarche and menopause, the more the risk increases

13. Which strategy should the nurse employ while teaching chemotherapy to a cancer patient?
A. Inform the patient that chemotherapy-related alopecia is typically permanent but that it can be treated with
long-term usage and wigs
B. To avoid unnecessary worrying, leave out the part wherein there are side effects
C. Inform the patient that the side effects of chemotherapy are unpleasant but never caused any death
D. Explain that antiemetics, antidiarrheals, and analgesics will be given as needed to alleviate adverse
effects [SG]
- Immunotherapy uses normal components of the immune system and is used therapeutically to boost or
manipulate the immune system to create an environment not conducive for cancer cell growth or that
attacks cancer cells directly. Immunotherapies may cause flu-like symptoms. The other options are not
correct.

14. Which of the following is an event in carcinogenesis?


A. Indiscriminate and continuous proliferation with loss of contact inhibition [SG] (Ratio)
- Malignant cells proliferate indiscriminately and continuously and lose the characteristic of contact
inhibition, growing on top of and in between normal cells. Cancer cells usually do not proliferate at a faster
rate than normal cells, nor can cell cycles be skipped in proliferation. However, malignant proliferation is
continuous, unlike normal cells.
B. Shortened phases of cell life cycles with occasional skipping of G1 or S phases
C. Rearrangement of stem cell RNA that causes abnormal cellular protein synthesis
D. A rate of cell proliferation that is more rapid than that of normal body cells

15. Which of the following is done as prophylaxis to breast cancer in patient with BRCA mutations?
A. Oopherectomy (Ratio)
B. Hysterectomy
C. Estrogen hormone replacement
D. HPV vaccination
E. Bilateral mastectomy (Ratio)

16. A post-gastrectomy gastric CA patient is manifesting a HR of 114, orthostatic hypotension and diaphoresis after a
meal. The nurse suspects this is because of [INCOMPLETE]
A. Nothing, it is an expected finding
B. Drinking fruit shakes and eating crackers (Ratio)
C. Insulin administration
D. Vagal stimulation
17. Which of the following statements describe promotion in cancer development?
a. The removal of cancer-promoting variables reduces the likelihood of cancer formation [SG]
b. The stage is distinguished by rapid development and metastasis
c. The stage of cancer growth in which there is an irreversible change in the cell’s DNA is known as promotion
d. Obesity is one such promoting factor [SG]
e. Tobacco smoke is a complete carcinogen capable of both initiation and progression [SG]
Rationale: Promoting factors, such as obesity and tobacco smoke promote cancer in the promotion stage of cancer
development. Eliminating risk factors can reduce the chance of cancer development as the activity of promoters is
reversible in this stage. Increased growth, invasion, and metastasis are seen in the progressive stage.

18. A patient with lung cancer manifests with headache, general fatigue and memory loss, the nurse knows that this
syndrome is indicative of a [INCOMPLETE]
a. Thrombosis
b. Paraneoplastic syndrome
c. Superior vena cava syndrome
d. Dumping syndrome

19. Which of the following is NOT a characteristic of malignant cells?


A. Bears little resemblance to parent cell
B. Metabolic activity is simplified
C. Performs typical cell function
D. Unregulated rate of mitosis

20. Which of the following serologic tests will lead the nurse to suspect that a patient with right-sided abdominal pain may
have liver cancer?
a. Elevated creatinine
b. Elevated serum phosphorus levels
c. Elevated CA-125
d. Elevated serum-fetoprotein (AFP) levels (Ratio)

21. A patient with ovarian cancer manifests abdominal distention, shiny abdominal skin and dyspnea. The nurse prepare
the patient for a procedure called
a. Hysterectomy
b. NGT insertion
c. Paracentesis
d. Exploratory laparotomy

22. Nurse is providing health teachings regarding the ways preventing ovarian cancer. Which of the following statements
indicates the need for further teaching?
a. To decrease risk of ovarian cancer, plan to breastfeed if you have children
b. To decrease risk of ovarian cancer, avoid using oral contraceptives for birth control (Ratio)
c. To decrease risk of ovarian cancer, decrease the fat in your diet
d. To decrease risk of ovarian cancer, bear children if physically and psychologically able

23. Which of the following statements best describes “carcinoma in situ.”?


a. Antigens on the surface of tumor cells that elicit an immune response
b. Cancer cells that evade the immune system
c. Capable of generative cancer-related cellular changes
d. Except for invasion, the lesion has histologic characteristics of cancer [SG] (Ratio)
- Carcinoma in situ has the histologic features of cancer except invasion. Evasion of the immune system by
cancer cells by various methods is immunologic escape. Oncogenic factors can cause cellular alterations
associated with cancer. Tumor cell surface antigens that stimulate an immune response are
tumor-associated antigens.

24. A nurse is caring for a client with gastric cancer. Which of the following indicates metastasis to the peritoneal cavity?
a. Presence of ascites
b. The client is reporting rapid weight loss
c. A nurse observes Grey Turner’s sign
d. The client is reporting nausea

25. A patient with ovarian cancer manifests with symptoms of hyperactive bowel sounds, projectile vomiting, and
abdominal pain. The nurse prepares to:
a. Insert a gastric lavage (Ratio)
b. NGT gavage
c. Paracentesis
d. Hysterectomy
26. Which of the following is the most common cause of bleeding among cancer patients?
a. Hypoxemia
b. Anemia
c. Coagulation disorders
d. Decreased platelet count (Ratio)

27. Which of the following best describes the use of tamoxifen (Nolvadex) in breast cancer
a. Because tamoxifen has been shown to increase the risk for uterine cancer, it is only used when other treatment
has not been successful.

b. Tamoxifen is used only to prevent the development of new primary tumors in women with high risk for breast
cancer
c. Tamoxifen is the primary treatment for breast cancer if axillary lymph nodes are pistive for cancer.
d. Tamoxifen is the treatment of choice after surgery if the tumor has receptors for estrogen and
progesterone on its cells. [SG] (Ratio)
- Tamoxifen is an antiestrogen agent that blocks the estrogen- receptor sites of malignant cells and is the
usual first choice of treatment in women with hormone receptor–positive tumors, with or without nodal
involvement. Tamoxifen reduces the risk for recurrent breast cancer and new primary tumors. The side
effects of the drug are minimal and are those associated with decreased estrogen.

28. A nurse is caring for a patient with metastatic liver cancer. The nurse knows that:
a. Metastatic cancer of the liver is more responsive to treatment than primary carcinoma of the liver
b. The patient will undergo surgery to remove the involved portions of the liver.
c. Chemotherapy is highly successful in the treatment of liver cancer.
d. Supportive care that is appropriate for all patients with severe liver damage is indicated. [SG] (Ratio)
- Because the prognosis for cancer of the liver is poor and treatment is largely palliative, supportive nursing
care is appropriate. The patient exhibits clinical manifestations of liver failure, as seen in any patient with
advanced liver failure. Whether the cancer is primary or metastatic, there is usually a poor response to
chemotherapy and surgery is indicated in the few patients that have localization of the tumor when there
is no evidence of invasion of hepatic blood vessels.

29. A woman receiving a modified radical mastectomy for breast cancer will have lymphatic mapping with sentinel lymph
node dissection (SLND). What information does the nurse impart to the patient and her family?
a. Lymphatic mapping with SLND provides metastatic lymph nodes to test for responsiveness to chemotherapy.
b. If malignant cells are found in any sentinel nodes, a complete axillary lymph node dissection will be done.
c. If one sentinel lymph node is positive for malignant cells, all of the sentinel lymph nodes will be removed.
d. Lymphatic mapping indicates which lymph nodes are most likely to have metastasis and all of those
nodes are removed. [SG] (Ratio)
- In a sentinel lymph node biopsy (SLNB) radioisotopes or dye identify lymph nodes that drain from the
tumor site, and they are removed. Those nodes are examined for malignant cells. If any of the nodes
have malignant cells, the next step is a complete axillary lymph node dissection (ALND). If the sentinel
nodes are negative, no other lymph nodes are removed.

30. Which of the following interventions will prevent Deep Vein Thrombosis in ovarian cancer patients post hysterectomy?
a. Increase hydration
b. Use of antiembolic stockings
c. Counting the perineal pads
d. Complete bed rest without toilet privileges

31. A nurse is caring for a patient with undergoing radiation therapy for cervical cancer. The patient is manifesting
diarrhea. Which of the following advice will be appropriate for this client?
a. Drink sodium-rich drinks and apply hydrocolloid dressings to inflamed regions.
b. Advise a low-residue diet. (Ratio)
c. Increase your milk consumption and take more showers.
d. Drink potassium-free drinks and soak in the tub on a regular basis.
32. Which of the following are prophylactic measures against ovarian cancer?
a. salpingectomy and oopherectomy [TRANS]
b. safe sexual practices
c. use of oral contraceptives [TRANS]
d. delaying sexual debut
e. bilateral mastectomy

33. Which of the following is NOT a characteristic of a benign tumor?


a. overcomes contact inhibition
b. responds to body's homeostatic controls
c. frequently encapsulated
d. well-differentiated cell

34. A nurse is caring for a patient with epigastric pain and is suspected for gastric cancer. The nurse will prepare the
patient for which diagnostic procedure?
a. Bronchoscopy
b. Colonoscopy
c. Esophagogastroduodenoscopy
d. Arthroscopy

35. In which of the following situations will the nurse recommend “watchful waiting” in a patient with prostate cancer?
a. When the client is older than age 70 with a life expectancy of less than 10 years with low-grade disease
(Ratio)
b. When a client has extension of the tumor outside of the prostate
c. When a client has an elevated prostate specific antigen, has no symptoms, and is under the age of 60
d. When bone cancer is diagnosed along with prostate cancer (klev)

36. A post-mastectomy patient is reporting sharp, prickly feelings in her right arm. The nurse knows that this is a post-op
complication called
a. Lymphedema
b. Phantom Breast Pain
c. None of the choices
d. Post-Breast Pain Therapy Syndrome

37. Which type of hepatitis is a DNA virus, can be transmitted via exposure to infectious blood or body fluids, is required
for HDV to replicate, and increases the risk of the chronic carrier for hepatocellular cancer?
a. Hepatitis C (HCV)
b. Hepatitis A (HAV)
c. Hepatitis B (HBV)
d. Hepatitis E (HEV)

38. In which of the following organs does colorectal cancer usually metastasizes?
a. Liver
b. Cervix
c. Bone
d. Brain

39. Which of the following is NOT a characteristic of a malignant tumor?


a. exhibits cell specialisation
b. cells lacking differentiation
c. exhibits angiogenesis
d. uncontrolled growth causes death

40. Which is FALSE regarding the normal cell proliferation?


a. The rate of cell proliferation in the myocardium and cartilage is slow.
b. Cell proliferation occurs when there is a physiologic need for the body for more cells.
c. Normal cells do not exhibit contact inhibition.
d. Cell proliferation is activated when there is cell death.

41. Which of the following symptoms in lung cancer may indicate that the tumor is growing in the esophagus
a. Seizure attack
b. Dysphagia
c. Lymphadenopathy
d. Pleuritic pain

42. Which of the following molecule/structure is an indication of the shift of the cells to more immature metabolic pathway
a. Oncofetal antigens
b. Protooncogence
c. Oncogenes
d. Tumor suppressor genes

43. Which vaccine should a nurse recommend for prevention of liver cancer
a. Hepatitis A vaccine
b. Varicella vaccine
c. Meningococcal vaccine
d. Hepatitis B vaccine (Ratio)

44. In which of the following situation in cases of ovarian cancer will the nurse prepare to administer Enoxaparine
(Clexane)?
a. The patient cannot sleep
b. The patient is having dull abdominal pain
c. The patient is having a distended abdomen
d. The patient manifests calf pain and tenderness (Ratio)

45. The following medical interventions are for patients with lung cancer undergoing palliation therapy, EXCEPT:
a. Radiation therapy
b. Administration of painkillers
c. Wedge resection of the lungs (Ratio)
d. Bronchial stenting

46. A patient is to undergo a pelvic exam, the nurse


a. Positions the patient in lithotomy
b. Exposes the hips of the patient
c. Does skin preparation on the patient's abdomen
d. Prepares the fine needle for aspiration

47. Which of the following are risk factors for cervical cancer?
a. Sex with uncircumcised men
b. BMI >30 (klev)
c. Exposure to secondhand smoke
d. Having sex with men infected with HPV
e. Late childbearing

48. Which of the following symptoms indicate poor prognosis in gastric cancer?
a. Abdominal distention with abdominal pain
b. Feelings of early satiety
c. Indigestion
d. Anorexia and ulcer-like pain

49. Which of the following symptoms of lung cancer may indicate airway obstruction?
a. Hoarseness of voice
b. Tachycardia
c. High pitched sound upon auscultation
d. Pleuritic pain

50. Which of the following will you teach a client whose father died of prostate cancer?
a. Consuming fresh fruits and vegetables instead of high-fat diets may reduce the risk of prostate cancer
(Tom)
b. Prostate cancer can be prevented by treating any enlargement of the prostate gland
c. The use of natural herb, saw palmetto, has been shown to be useful in the prevention of prostate cancer
d. Nothing can reduce the risk because prostate cancer is predominantly an aging illness.

51. A patient post-mastectomy feels still feels pain on the breast. The nurse knows that this is a post-op complication
called
a. Paraneoplastic syndrome
b. Post-breast therapy pain syndrome
c. Lymphedema
d. None of the choices

52. Which of the following are preventive measures against cervical cancer?
a. Regular pelvic pap tests
b. Regular transvaginal ultrasounds
c. HPV immunization
d. Advising delay in sexual debut
e. Practicing safe sex

53. After a thyroid cancer patient undergoes subtotal thyroidectomy. The nurse caring for this patient immediately asks the
patient to state her name as soon as she wakes up. Which of the following is the nurse trying to check?
a. Internal hemorrhage.
b. Decreasing level of consciousness.
c. Laryngeal nerve damage. (Ratio)
d. Upper airway obstruction.

54. A patient is being scheduled for a digital rectal exam. Which of the following is indicative of prostate cancer?
a. A nonindurated prostate
b. A boggy, tender prostate.
c. Abdominal pain.
d. A hard prostate, localized or diffuse. (pakyang) (Tom)(gen)

55. A client is about to undergo a total prostatectomy and asks about the affectation of this surgery to sexual function.
Which of the following statements provides a correct explanation to the patient?
A. "Loss of the prostate gland indicates no loss of sexual function or urge."
B. "The loss of the prostate gland indicates that your erectile capacity will return shortly following surgery."
C. "Initially, erectile capability is disrupted but will gradually return over the year.” (Ratio)
D. "Loss of the prostate gland indicates you'll be impotent."

56. The patient with prostate cancer is scheduled for a bilateral orchiectomy. He inquires about the procedure. The nurses
responds correctly by saying that a bilateral orchiectomy
A. “Freezes prostate tissue, causing cell death.”
B. “Causes the likelihood of urine incontinence and impotence.”
C. “Is when the whole prostate gland, prostatic capsule, and seminal vesicles are removed."
D. “Causes a decrease in the principal circulating substances testosterone, androgen.” (Ratio)

57. Which of the following cancer type is associated with excessive fat in the diet?
A. Liver
B. Colon
C. Lung
D. Ovarian.

58. A patient with cervical cancer is admitted with a radiation implant. Which of the following should the nurse prioritize?
a. Encourage the family to visit.
b. Encourage the client to take frequent rest periods.
c. Admit the client to a private room. (Ratio)
d. Place the client on protective isolation.

59. Which of the following statements accurately describes the subtotal gastrectomy (Billroth II procedure)?
a. The vagus nerve will be repositioned
b. The pyloric sphincter will expand
c. The duodenum will be removed as part of the surgery.
d. The gastric stump will be anastomosed to the jejunum

60. What gastrointestinal alterations causes dumping syndrome in a gastric cancer patient who underwent gastrectomy?
a. The liver produces an excessive amount of glycogen.
b. The contents of the stomach are quickly emptied into the small intestine.
c. Excessive digestive enzyme production in the intestines.
d. Gastric enzyme deficiency

61. A nurse is caring for a client with metastatic lung cancer. The nurse assess for which manifestation?
a. Diarrhea
b. Weight gain
c. Constipation
d. Hoarseness
62. The nurse is caring for a client with thyroid cancer who underwent a subtotal thyroidectomy. Which of the following is
indicative of tetany?
a. Tingling in the fingers
b. Pains in the joints of the hands and feet.
c. Bleeding on the back of the dressing.
d. Tension on the suture line.

63. A nurse is caring for a patient who underwent a radical mastectomy with axillary node dissection. The patient inquires
about lymphedema. The nurse correctly responds that

a. “Lympedema occurs when all cancer cells are not removed."


b. At any time after surgery or not at all." (Cam) (Tom) (pakyang)
- Lymphedema can happen any time after surgery or radiation to the lymph nodes
c. In older women.
d. “Only with radical mastectomy."

64. Which of the following medications should be available in the E-cart when you are caring for a thyroid cancer client
who underwent subtotal thyroidectomy?
a. Sodium phosphate
b. Calcium gluconate
c. Sodium bicarbonate
d. Echothiophate iodide

MEDICAL-SURGICAL 2 NURSING SAMPLEX


Exam Compilation
UNIT EXAM 1 (MEDICAL-SURGICAL 2)

1. IBD is a common inflammatory functional bowel disorder also known as spastic bowel, functional colitis and mucous
colitis. Which of the following factors causes ulcerative colitis?
a. Emotional stress
b. Chronic constipation (hunch, since constipation in prolonged periods can cause ulcerative colitis)
c. Altered immunity (wrong to sir, but is the correct answer)
- The cause is unknown but is thought to be multifactorial. Heredity, infectious agents, altered immunity or
autoimmune and environmental factors are considered to cause Crohn’s disease and ulcerative colitis.
d. Acidic diet

2. Which of the following diets would be most appropriate for the client with ulcerative colitis?
a. Low fat, high fiber
b. High calorie, low protein
c. High protein, low residue
d. Low sodium, high carbohydrates

3. A female client who has just been diagnosed with hepatitis A asks, “How could I have gotten this disease?” WHat is the
nurse’s best response?
a. You must have received an infected blood transfusion
b. You may have eaten contaminated restaurant food
c. You could have gotten it by using IV drugs
d. You probably got it by engaging in unprotected sex

4. The nurse understands that the best position for the client that has undergone a gastrectomy is:
a. Low Fowler’s (wrong to sir, but is the correct answer)
- A client who has had abdominal surgery is best placed in a low Fowler’s position postoperatively. This positioning
relaxes abdominal muscles and provides for maximum respiratory and cardiovascular function. The prone,
supine, or Sims position would not be tolerated by a client who has had abdominal surgery, nor do those positions
support respiratory or cardiovascular functioning.
b. Right or left Sim’s
c. Prone
d. Supine [SIR P, to prevent irritation of the incision site of the gastrectomy]]

5. Mrs. Cruz was admitted to the Blue Station due to pyrosis (heartburn), dyspepsia and difficulty of swallowing. What
diagnostic test would confirm the type of problem Mrs. Cruz have?
a. Barium swallow
b. Lower GI series
c. Colonoscopy
d. Barium enema

6. A nurse would assess a 3-year-old child who has Hirschsprung’s disease for:
a. Abdominal distention and ribbon-like stools
b. Tight rectal sphincter and watery stools
c. Prolapse rectum and mucous stools
d. Periumbilical pain and clay-colored stools

7. Mr. Crisostomo is admitted to the hospital with a bowel obstruction. He complained of colicky pain and inability to pass
stool. Which of these findings by Nurse Leonard would indicate that the obstruction is in the early stages?
a. Hypoactive bowel sounds [hunch, since obstruction man so basin automatic hypoactive kang Sir P]
b. Normal bowel sounds heard in all four quadrants
c. No bowel sounds auscultated
d. High pitched tinkling or rumbling sounds (wrong to sir, but is the correct answer)
- Early in the bowel obstruction, the bowel attempts to move the contents past the obstruction and this is heard as
high pitched tinkling bowel sounds. As the obstruction progresses, bowel sounds will diminish and may finally
become absent.

8. Nurse David is the staff nurse assigned at the Emergency Department. During this shift, a patient was rushed - in the
emergency department complaining of severe heartburn, vomiting and pain that radiates to the flank. The patient is for
occult blood examination. What specimen will you collect?
a. Gastric juice
b. Stool
c. Blood
d. Urine

9. A client describes abdominal discomfort following the ingestion of milk. The nurse recognize that this may be the result
of a genetic deficiency of the enzymes.
a. Amylase
b. Lactase
c. Maltase
d. Sucrase

10. A 6-month old infant is suspected of having intussusception. A nurse should expect the child to undergo which of the
following procedures.
a. Cholangiography
b. Rectal biopsy
c. Colonoscopy
d. Barium enema

11. Mrs. Cruz was admitted to the Blue Station with complaints of difficulty swallowing. This term is referred to as:
a. Pyrosis
b. Dysphagia
c. Dyspepsia
d. Odynophagia

12. The client with liver cirrhosis would have which of the following laboratory results?
a. Increased serum magnesium levels
b. Increase serum albumin
c. Normal prothrombin time
d. Elevated SGOT (serum glutamic-oxaloacetic transaminase)

13. Nurse David is the staff nurse assigned at the emergency department. During this shift, a patient was rushed - in the
emergency department complaining of severe heartburn, vomiting and pain that radiates to the flank. What diagnostic test
would yield good visualization of the ulcer crater?
a. Gastroscopy
b. Barium swallow
c. Histology
d. Endoscopy

14. A client admitted with anorexia, weight loss, abdominal distention and abnormal stools. A diagnosis of malabsorption
syndrome is made. To meet the client’s needs, the nurse should
a. Institute IV therapy to improve hydration
b. Allow the client’s need to eat food preferences
c. Maintain NPO status because food precipitates diarrhea
d. Encourage consumption of meats at a meal time and high protein
- The diet should be high in protein and calories, low in fat, and gluten-free for individuals with malabsorption
syndrome . Protein is needed for tissue rebuilding. The client may prefer foods high in gluten, which will potentiate
malabsorption. IV therapy is a dependent function and does not provide all the necessary nutrients. Diarrhea is
caused by malabsorption, which accounts for the depressed nutritional status; once the diarrhea is corrected, it is
essential to compensate by providing a nutritious diet.

15. Which of the following would the nurse include in the teaching plan as the most common symptom of colon cancer?
a. Abdominal distension
b. Diarrhea
c. Abdominal pain
d. Rectal bleeding

16. Manny, 6-years old was admitted to the General Hospital due to increasing frequency of bowel movements, abdominal
cramps and distention. What life-threatening condition may result in persistent diarrhea?
a. Leukocytosis
b. Dehydration [hunch, most common complication sa hospital]
c. Hypokalemia
d. Cardiac dysrhythmias (wrong to sir, but is the correct answer)
- Due to increased frequency and fluid content in the stools, diarrhea may cause fluid and electrolyte imbalance
such as hypokalemia. Once potassium is depleted, this will affect the contractility of the heart causing cardiac
arrhythmia leading to death.

17. A female client with hepatitis C develops liver failure and GI hemorrhage. The blood products that would likely bring
about hemostasis in the client are:
a. Fresh frozen plasma and whole blood
b. Whole blood and albumin
c. Platelets and packed red blood cells
d. Cryoprecipitate and fresh frozen plasma (wrong to sir, but is the correct answer)
- The liver is vital in the synthesis of clotting factors, so when it's diseased or dysfunctional, as in hepatitis C,
bleeding occurs. Treatment consists of administering blood products that aid clotting. These include fresh frozen
plasma containing fibrinogen and cryoprecipitate, which has most of the clotting factors. Although administering
whole blood, albumin, and packed cells will contribute to hemostasis, those products aren't specifically used to
treat hemostasis. Platelets are helpful, but the best answer is cryoprecipitate and fresh frozen plasma.

18. Which of the following would be an expected outcome for a client with peptic ulcer disease?
a. The client demonstrate the appropriate use of analgesics to control pain
b. The client will explain the rationale for eliminating alcohol from the diet
c. The client will eliminate contact sports from his or her lifestyle
d. The client will verbalize the importance for monitoring hemoglobin and hematocrit every 3 months

19. A client is taking antacid for treatment of peptic ulcer. Which of the following statements best indicates that the client
understands how to correctly take the antacid?
a. “I should take my antacid before I take my other medications”
b. “It is best for me to take my antacid 1-3 hours after a meal” (wrong to sir, but is the correct answer)
- Antacids are most effective if taken 1 to 3 hours after meals and at bedtime. When an antacid is taken on an
empty stomach, the duration of the drug’s action is greatly decreased. Taking antacids 1 to 3 hours after a meal
lengthens the duration of action, thus increasing the therapeutic action of the drug. Antacids should be
administered about 2 hours after other medications to decrease the chance of drug interactions. It is not
necessary to decrease fluid intake when taking antacids.
c. “My antacid will be most effective if I take it whenever I experience stomach pain”
d. “I need to decrease my intake of fluids so that I don’t dilute the effects of my antacid”

20. Which of the following explanations should a nurse give to a client regarding the primary cause of peptic ulcer
disease?
a. “Infection with Helicobacter pylori causes ulcers”
b. “Executive job positions predispose people to ulcer formation”
c. “Seasonal changes are associated with ulcer disease”
d. “Spicy diet contribute to ulcer development”

21. Which of the following would be an expected nutritional outcome for a client who has undergone a subtotal
gastrectomy for cancer?
a. Control nausea and vomiting through regular use of antiemetics
b. Achieve normal nutritional status through oral and parenteral feedings
c. Resume normal dietary intake of 3 meals a day
d. Regain weight loss after 1 month after surgery

22. When planning dietary teaching for a client with malabsorption syndrome, the nurse should include the need to avoid?
a. Fruit juice
b. Rye bread (Sir P, because of gluten)
c. Corn
d. Cheese

23. The alimentary canal is a continuous, coiled hollow muscular tube that winds through the ventral cavity and is open at
both ends. It solid organs include all of the following except:
a. Stomach (wrong to sir, but is the correct answer)
- Stomach is a hollow digestive organ in the GI tract. The liver, gallbladder and pancreas are all solid organs which
are part of the hepato-biliary system.
b. Pancreas
c. Liver
d. Lungs [hunch, since mao ra ang dili organ nga part sa digestive]

24. The nurse would regularly assess the client’s ability to metabolize the TPN solution adequately by monitoring the client
for which of the following signs?
a. Hypertension
b. Elevated BUN concentrations
c. Tachycardia
d. Hyperglycemia

25. A client who has been diagnosed with GERD complains of heartburn. To decrease the heartburn, the nurse should
instruct the client to eliminate which of the following items from the diet?
a. Raw vegetables
b. Lean beef
c. Air popped corn
d. Hot chocolate

26. Mr. Gerald Liu 19 years old is being admitted to a hospital unit complaining of severe pain in the lower abdomen.
Admission vital signs reveal an oral temperature of 101.2F. Which of the following would confirm a diagnosis of
appendicitis?
a. Mr. Liu describes the pain as occurring 2 hours after eating
b. The pain is localized at a position halfway between the umbilicus and the right iliac crest
c. The pain subsides after eating
d. The pain is in the left lower quadrant

27. The doctor ordered a complete blood count. After the test, Nurse Ray received the result from the laboratory. Which
laboratory values confirm the diagnosis of appendicitis?
a. RBC 5.5 x 106/mm3
b. Hgb 15g/dL
c. WBC 13000/mm3
d. HCT 44%

28. A client with a gastric ulcer is advised by his physician not to drink coffee. The nurse explains that coffee
a. Increases mental stress
b. Increase smooth muscle tone
c. Stimulate gastric secretions
d. Elevates systolic blood pressure.

29. Manny, 6-years old was admitted to the General Hospital due to increasing frequency of bowel movements, abdominal
cramps and distention. Diarrhea is said to be the leading cause of morbidity and mortality in the Philippines. Nurse Harry
knows that diarrhea is present if:
a. Passage of stool is more than 3 bowel movements per day
b. Passage of stool is less than 3 bowel movements per week
c. Passage of stool is less than 3 bowel movements per day
d. Passage of stool is more than 3 bowel movements per week
30. Which of the following complications is thought to be the most common cause of appendicitis?
a. Abdominal wall swelling
b. A fecalith
c. Bowel kinking
d. Internal bowel occlusion

31. Most digestive activity occurs in the pyloric region of the stomach. What hormone stimulates the chief cells to produce
pepsinogen?
a. Gastrin
b. Insulin
c. Pepsin
d. HCl

32. Nurse Ivan instructs her client who has had a hemorrhoidectomy not to use a sitz bath until at least 12 hours
postoperatively to avoid which of the following complications?
a. Hemorrhage
b. Rectal spasm
c. Urinary retention
d. Constipation

33. A client who has had a total gastrectomy is given instructions on measures to prevent the development of dumping
syndrome. Which of the following statements, if made by the client would indicate a correct understanding of the
instructions?
a. “I will only include high-fiber foods on my diet”
b. “I will rest one hour before each meal”
c. “I will have a bedtime snack”
d. “I will avoid concentrated sugars” [Sir P, sugar pulls water away from cells causing dumping syndrome]

34. What pancreatic enzyme aids in the digestion of carbohydrates?


a. Pepsin
b. Lipase
c. Chymotrypsin
d. Amylase

35. What is the immediate home care management for diarrhea?


a. Water
b. Milk
c. Imodium
d. Oresol

36. Which of the following symptoms would be indicative of the dumping syndrome?
a. Heartburn
b. Hunger
c. Vomiting
d. Diaphoresis

37. A male client with cholelithiasis has a gallstone lodged in the common bile duct. When assessing this client, the nurse
expects to note:
a. Black, tarry stools
b. Yellow sclera
c. Light amber urine
d. Circumoral pallor

38. A client is diagnosed with malabsorption syndrome. Striking clinical improvement should be noted after administration
of:
a. Vitamin B12
b. Folic acid
c. Corticotropin
d. A gluten-free diet

39. The nurse explained to the client that some form of skin barrier must be used around the stoma at all times. The
primary function of a skin barrier is to:
a. Allow the client to keep the ostomy pouch on longer
b. Protect against irritation from ileostomy effluent
c. Help maintain an accurate output record
d. Help prevent the formation of odor

40. Mr. Gerald Liu 19 years old is being admitted to a hospital unit complaining of severe pain in the lower abdomen.
Admission vital signs reveal an oral temperature of 101.2F. After a few minutes, the pain suddenly stops without any
intervention. Nurse Ray might suspect that
a. Signs and symptoms of peritonitis occur
b. The appendix is still distended
c. An increase in intrathoracic pressure will occur
d. The appendix may have ruptured

41. Peptic ulcer disease particularly gastric ulcer is thought to be caused by which of the following microorganisms?
a. H.pylori
b. E.coli
c. S.aureus
d. K.pneumoniae

42. A nurse is conducting discharge teaching for a client with Hepatitis B. Which statement by the client indicates that he
understands the teaching?
a. I can safely give blood after 3 months
b. Now I can never get hepatitis again
c. My family knows that if I got tired and start vomiting, I may be getting sick again
d. I’ll never have a problem with my liver again, even if I drink alcohol

43. If a client has an acute appendicitis, the report of the laboratory tests would most likely show which of these results?
a. Decrease serum potassium levels
b. Elevated leukocyte count
c. Low hemoglobin levels
d. Elevated erythrocyte level

44. Mrs. Cruz was admitted to the Blue Station due to pyrosis (heartburn), dyspepsia and difficulty of swallowing. Based
on the symptoms presented, Nurse Reynand might suspect:
a. GERD
b. Esophagitis
c. Gastric ulcer
d. Hiatal hernia

45. A nurse is caring for a client with hepatic encephalopathy. The nurse expects which of the following laboratory values
to be abnormal?
a. Amylase
b. Potassium
c. Ammonia
d. Calcium

46. Daily abdominal girth measurements are prescribed for a client with liver dysfunction and ascites. To increase
accuracy, the nurse should use which landmark?
a. Xiphoid process
b. Symphysis pubis
c. Iliac crest
d. Umbilicus

47. The nurse is reviewing the record of a male client postoperatively following a creation of a colostomy. Which nursing
diagnosis should the nurse include in the plan of care?
a. Fear related to poor prognosis
b. Sexual dysfunction
c. Body image, disturbed
d. Nutrition more than body requirements, imbalanced

48. A nurse would expect a client with ulcerative colitis to report which of the following manifestations of the disease?
a. Esophageal reflux
b. Bloody diarrhea
c. Flank pain
d. Abdominal distention

49. Diet that will be followed or a patient with peptic ulcer after discharge will consist of the following?
a. High-protein foods
b. Large amount of milk
c. Any foods that will be tolerated
d. Bland foods

50. A nurse would instruct a client who has had an ileostomy to AVOID which of the following foods?
a. Potatoes
b. Bread
c. Beef [hunch, maoy pinakagahi nga food among the rest]
d. Popcorn and egg (wrong to sir, but is the correct answer)
- Popcorn is difficult to digest and could block the stoma. The patient should either consume these foods in very
small amounts or avoid them altogether.

51. A client has had a total gastrectomy. Which of the following instructions should a nurse give to the client on avoiding
dumping syndrome?
a. Include complex carbohydrates with your meals
b. Eat three balanced meals per day
c. Add polyunsaturated fats to your daily meals
d. Limit fluid intake with your meals

52. The nurse caring for a client with small-bowel obstruction would plan to implement which nursing intervention first?
a. Preparing to insert a nasogastric tube
b. Administering pain medication
c. Obtaining a blood sample for laboratory studies
d. Administering IV fluids
- The nurse should first administer I.V. infusions containing normal saline solution and potassium to maintain fluid
and electrolyte balance. For the client's comfort and to assist in bowel decompression, the nurse should prepare
to insert an NG tube next. A blood sample is then obtained for laboratory studies to help diagnose bowel
obstruction and guide treatment. Blood studies usually include a complete blood count, serum electrolyte levels,
and blood urea nitrogen level. Pain medication commonly is withheld until obstruction is diagnosed because
analgesics can decrease intestinal motility.

53. Which of the following nursing diagnoses would be the most appropriate for a client with intestinal obstruction?
a. Impaired swallowing r/t NPO status
b. Chronic Pain r/t abdominal distention
c. Deficient fluid volume r/t nausea and vomiting
d. Urinary retention r/t deficient fluid volume

54. Which of the following would be the expected nutritional outcome for a client who has undergone a subtotal
gastrectomy for cancer?
a. Regain weight loss after 1 month after surgery
b. Achieve nutritional status through oral or parenteral feedings
c. Control nausea and vomiting through regular use of antiemetics
d. Resume normal dietary intake of three meals a day

55. A client is scheduled to have an upper GI series. Which of the following treatments should the nurse anticipate after
examination?
a. Placing the client on clear liquid diet
- The patient should be placed on a low-residue diet for 2 to 3 days prior the upper gastrointestinal series.
b. Giving the client a tap water enema
c. Starting an IV infusion
d. Administering laxative [mao ni ako mali sa exam, pero sa ratio ni sir is mag laxative para maexpel ang
barium sulfate, so basin namali ra siya ug input sa neo]

56. A client’s ulcerative colitis symptoms have been present for longer than 1 week. The nurse recognizes that the client
should be assessed carefully for signs of which of the following complications?
a. Heart failure
b. Hypokalemia
c. Deep vein thrombosis
d. Hypocalcemia

57. What laboratory finding is the primary diagnostic indicator for pancreatitis?
a. Increased lactate dehydrogenase
b. Elevated blood urea nitrogen (BUN)
c. Elevated serum lipase
d. Elevated aspartate aminotransferase

58. Which of the following has been identified as a potential risk factor for the development of colon cancer?
a. Long term use of laxative
b. History of inflammatory bowel disease
c. History of smoking
d. Chronic constipation

59. Which of the following would be a priority focus of care for a client experiencing an exacerbation of his Crohn’s
disease?
a. Maintaining current weight
b. Decreasing episodes of rectal bleeding
c. Encouraging regular ambulation
d. Promoting bowel rest (wrong to sir, but is the correct answer)
- A priority goal of care during an acute exacerbation of Crohn's disease is to promote bowel rest. This is
accomplished through decreasing activity, encouraging rest, and initially placing clients on nothing-by-mouth
status while maintaining nutritional needs parenterally. Regular ambulation is important, but the priority is bowel
rest. The client will probably lose some weight during the acute phase of the illness. Diarrhea is nonbloody in
Crohn's disease, and episodes of rectal bleeding are not expected.

60. A client with cirrhosis of the liver develops ascites. The nurse should expect the physician to write which of the
following orders?
a. High sodium diet
b. Restrict fluid to 1,000mL per day
c. Ambulate 100 ft, TID
d. Maalox 30mg PO BID

UNIT EXAM 2 (MEDICAL-SURGICAL NURSING 2)

1. The most common cause of hyperthyroidism is Graves’ disease. What type of disorder does it fall under?
a. Autoimmune
b. Infectious
c. Allergic
d. Genetic

2. A nurse develops a postoperative care plan for a client who is scheduled for hypophysectomy. What interventions
should be included in the strategic plan? Select all that apply.
a. Encourage coughing and deep breathing (avoid coughing)
b. Obtain daily weight
c. Elevate the head of the bed
d. Use a soft toothbrush for mouth care (avoid tooth brushing after surgery)
e. Monitor intake and output

3. The client with SIADH is being cared for by the nurse. What interventions should the nurse impose? Select all that
apply.
a. Monitor for hyperactive reflexes and heightened alertness
b. Prepare to give a 500-mL NaCl intravenous fluid bolus
c. Place the client on a fluid-restricted diet as prescribed
d. Administer furosemide intravenously as prescribed
e. Obtain the weight near the same time each morning

4. Which of the following actions does the nurse providing vasopressin (Pitressin) to a client with diabetes insipidus
anticipate performing this medication?
a. Maintaining NPO status
b. Administering IV hypertonic fluids
c. Monitoring urine output
d. Maintaining fluid restrictions

5. Which clinical change should indicate to a nurse that the therapy for a client with Addisonian crisis is effective?
a. An increase of 25 mm Hg in the client’s systolic blood pressure
- An increase in blood pressure is indicative of effective therapy. Addisonian crisis is caused by adrenocortical
insufficiency with disturbances of sodium and potassium metabolism resulting in hypernatremia and
hyperkalemia. The depletion of sodium and water causes severe dehydration and hypotension. Effective therapy
would lower potassium levels. A serum potassium level of 3.4 to 4.8 mEq/dL is within the normal range. The
normal serum calcium level is 8.6 to 10.2 mg/dL.
b. A decrease of 25 mm Hg in the client’s systolic blood pressure
c. An increase in the client’s serum potassium level from 3.4 to 4.8 mEq/dL
d. An increase in the client’s total serum calcium level from 8.6 to 10.0 mg/dL

6. Which of the following should a client Addisonian crisis prioritize?


a. Controlling hypertension.
b. Preventing irreversible shock.
- Addison's disease is caused by a deficiency of adrenal corticosteroids and can result in severe hypotension and
shock because of uncontrolled loss of sodium in the urine and impaired mineralocorticoid function. This results in
loss of extracellular fluid and dangerously low blood volume. Glucocorticoids must be administered to reverse
hypotension. Preventing infection is not an appropriate goal of care in this life-threatening situation. Relieving
anxiety is appropriate when the client's condition is stabilized, but the calm, competent demeanor of the
emergency department staff will be initially reassuring.
c. Preventing infection.
d. Relieving anxiety.

7. The nurse is admitting a patient who has recently had bilateral adrenalectomy. Which intervention does the nurse need
to include in the client’s treatment plan?
a. Discuss changes in body image
b. Consider occupational therapy
c. Avoid stress-producing situations
d. Preventing social isolation

8. The nurse attends after a patient who is being treated with fludrocortisone acetate an Addison's disease treatment.
When keeping track of the client's progress, what part of this medication's expected therapeutic benefit will the nurse
focus primarily on?
a. Stimulate thyrotropin production
b. Promote electrolyte balance
c. Stimulate thyroid production
d. Stimulate immune response

9. A client with diabetes mellitus is being taught about hypoglycemia by the nurse. Which statement by the client indicates
the need for further teaching?
a. "Hypoglycemia can strike at any time of day or night,"
b. “I must take my normal insulin as instructed if hypoglycemia arises."
c. "If hypoglycemia arises, I should consume 6 to 8 ounces of milk."
d. "If I'm sweating or trembling, I could be suffering from hypoglycemia."

10. NO PIC

11. The nurse is caring for a client who has been admitted to the hospital with Addisonian crisis. Which medication is
recommended?
a. Regular insulin
b. Hydrocortisone [Q&A, D]
- Addisonian crisis results from insufficient secretion of glucocorticoids and mineralocorticoids from the adrenal
cortex. Hydrocortisone is a corticosteroid used in the absence of sufficient glucocorticoid production.
Hypoglycemia, not hyperglycemia, is associated with Addisonian crisis. Ketoconazole is a systemic antifungal that
has an unlabeled use in treating Cushing’s syndrome as an adrenal enzyme inhibitor. Addisonian crisis produces
severe hypotension; sodium nitroprusside is used to treat severe hypertension.
c. Sodium nitroprusside
d. Ketoconazole

12. A client has a total serum calcium level of 7.5 mg/dL (1.88 mmol/L). What studies could be performed to detect low
calcium levels?
a. Save urine to measure 17-ketosteroids
b. Conduct a Trousseau’s sign test [MAAM MANTE]
c. Observe the color of skin
d. Palpate turgor of the skin

13. The client, who has type 2 diabetes, is being assessed by the nurse. Which of the following results indicate to the
nurse that the client is suffering from HHNS? Select all that apply.
a. Blood pressure of 90/42 mmHg
b. Serum osmolality 364 mOsm/kg
c. Urine output 500mL past 8 hours
d. Very dry mucous membranes
e. Serum osmolality 364mOsm/kg

14. The nurse is looking for signs of hyperkalemia in a client with Addison's disease. Which signs/symptoms of the
electrolyte imbalance should the nurse look for in this case?
a. Prolonged bleeding time
b. Cardiac dysrhythmias [MAAM MANTE]
c. Dry mucous membranes
d. Polyuria

15. Which advice should a nurse give to an older client when teaching them about general hygienic practices for foot and
nail care? SELECT ALL THAT APPLY.
a. Use over-the-counter preparations to treat ingrown nails.
b. Use commercial removers for corns or calluses
c. Apply lanolin or baby oil if dryness is noted along the feet.
d. Wear knee-high hose to prevent edema.
e. Pat the feet dry thoroughly after washing and dry well between toes.
f. Soak and wash your feet daily using cool water.

16. The nurse witnesses a colleague caring for a client who had a transsphenoidal hypophysectomy 12 hours prior. Which
activity would compel the intervention of the observing nurse?
a. Elevates the head of the client’s bed to 30 degrees
b. Places a cold washcloth over the client’s swollen eyes
c. Moisturizes the client’s oral mucous membranes
d. Gathers supplies to replace the bloody nasal packing

17. A diabetic patient is being tested for glycosylated hemoglobin. When defining the aim of the laboratory test, the nurse
says glycosylated hemoglobin is utilized for which of the following?
a. To calculate the amount of glucose in hemoglobin for the past 4 months
b. To determine the average serum glucose level for the previous 4 months
c. To check for anemia and adequacy of tissue perfusion from previous 4 months
d. To compare hemoglobin to glucose levels of an average of 4 months

18. The nurse is keeping an eye on a client who has been diagnosed with type 1 diabetes. A glycosylated hemoglobin
level of 10% was discovered in today's blood work. The nurse devises a teaching strategy based on the knowledge that
this outcome identifies which discovery?
a. A high value that indicates that the client is not managing blood glucose control very well
b. A value that does not offer information regarding the client’s management of the disease
c. A normal value that indicates that the client is managing blood glucose control well
d. A low value that indicates that the client is not managing blood glucose control very well

19. Lyn, who is 70 years old, has been taking corticosteroids on and off for many years to treat her Rheumatoid
arthritis.Corticosteroids in large doses can have a variety of physiological and psychological effects. Lyn developed
Cushing's illness. Which information should be included in the teaching session for this patient? SELECT ALL THAT
APPLY
a. Monitor for excessive weight gain
b. Hold the medication if side effects occur
c. Keep a list of medications and doses with you
d. Increase the sodium in your diet
e. Avoid exposure to infection
f. Keep a medical identification device with you

20. You examine a person who has recently been diagnosed with Hypothyroidism and enlargement of the thyroid gland
(goiter). What physiological process is responsible for this enlargement?
a. A compensatory effort to produce more TH
b. An increased dietary iodine intake
c. An excess of TH stimulates thyroid follicles
d. Tissue hypertrophy in response to increased TH

21. The client is admitted to the hospital with a possible diagnosis of Cushing's disease. If the diagnosis of Cushing's
syndrome is confirmed, what laboratory findings might the nurse expect? SELECT ALL THAT APPLY.
a. Elevated serum cortisol
b. Hypocalcemia
c. Hyperglycemia
d. Eosinophilia
e. Hypokalemia
f. Thrombocytopenia

22. The nurse looks after a patient who is being treated with fludrocortisone acetate Addison's disease treatment. When
keeping track of the client's progress, what part of this medication's expected therapeutic benefit will the nurse
concentrate on?
a. Stimulate thyrotropin production.
b. Promote electrolyte balance.
c. Stimulate thyroid production.
d. Stimulate the immune response.

23. A client who was previously well-controlled on glyburide has lately started taking insulin. Fasting blood glucose levels
of 180 to 200 mg/dL (10.28 – 11.42 mmol/L) were reported. Which of the medications listed in the client's file could be
contributing to blood glucose levels that are too high?
a. Ciprofloxacin hydrochloride
b. Cimetidine
c. Ranitidine
d. Prednisone

24. What causes ketosis in people with diabetes?


a. Fat metabolism
b. Insulin secretion
c. Protein metabolism
d. Acid retention

25. A client with diabetes mellitus receives discharge instructions from the nurse. The glycosylated hemoglobin (HbA1c)
level of the client is 10%. In what way should the nurse inform the client?
a. “Increase the amount of vegetables and water intake in your diet regimen”
b. “Change the time of day you exercise because it may cause hypoglycemia”
c. “Utilize a high-intensity exercise regimen and decrease carbohydrate consumption”
d. “Continue with the same diet and exercise regimen you are currently using”

26. The nurse is looking after a patient who is having bilateral adrenalectomy to treat an adrenal tumor. What kind of
information should the nurse provide to the patient about the post-surgery requirements?
a. “You will not need any special long-term treatment after surgery”
b. “You will need to take daily hormone replacement beginning after the surgery”
c. “You will need to undergo chemotherapy after surgery”
d. “You will need to wear an abdominal binder after surgery”

27. The nurse is looking after a client who is suffering from pheochromocytoma symptoms. Which of the following
interventions should be included in this client’s care plan?
a. Assist with ambulation at least three times a day
b. Administer nicardipine for hypertension
c. Offer distractions such as television or music
d. Encourage family and friends to visit often
28. Which of the following is the most reliable indicator of whether a client with Addison's disease is getting the proper
quantity of glucocorticoids replacement?
a. Anorexia
b. Daily weight
- Measuring daily weight is a reliable, objective way to monitor fluid balance. Rapid variations in weight reflect
changes in fluid volume, which suggests insufficient control of the disease and the need for more glucocorticoids
in the client with Addison's disease. Nurses should instruct clients taking oral steroids to weigh themselves daily
and to report any unusual weight loss or gain. Skin turgor testing does supply information about fluid status, but
daily weight monitoring is more reliable. Temperature is not a direct measurement of fluid balance. Thirst is a
nonspecific and very late sign of weight loss.
c. Thirst
d. Temperature

29. After a thyroidectomy, which of the following procedures is recommended?


a. Support the patient’s head and neck with pillows and sandbag
b. Place pillows under the patient’s shoulders
c. Raise the knee-gatch to 30 degrees
d. Keep the patient in high-Fowler’s position

30. The nurse is putting together a care plan for an elderly client who has been diagnosed with type 1 diabetes who is
simultaneously suffering from acute gastroenteritis. To avoid dehydration, keep your diet and keep fluid consumption up to
date, which action should the nurse plan to include?
a. Withholding all fluids until vomiting has ceased entirely for at least 4 hours
b. Encouraging the client to take 8-12 ounces of fluid every hour while awake
- Dehydration needs to be prevented in the client with type 1 diabetes mellitus because of the risk of diabetic
ketoacidosis (DKA). Small amounts of fluid may be tolerated, even when vomiting is present. The client should be
offered liquids containing both glucose and electrolytes. The diet should be advanced as tolerated and include a
minimum of 100 to 150 g of carbohydrates daily. Offering water only and maintaining liquids for 5 days will not
prevent dehydration but may promote it in this client
c. Maintaining a clear liquid diet for at least 5 days before advancing to solid foods
d. Offering only water until the client is able to tolerate solid foods

31. The nurse is preparing to care for Addision’s disease patient who is stable. When completing an examination, what
skin appearance can the nurse expect?
a. Puffy and butterfly-like rash
b. Very white, dry, and scaly
c. Bronzed and suntanned hue
d. Diaphoretic and cyanotic

32. The nurse is getting ready to look after four patients. What is the best order for the nurse to care for the client?
1. Client following a thyroidectomy who has hoarseness and an Sa02 of 86%
2. Client with hyperparathyroidism with serum calcium level of 10.1 mg/dL
3. Client with diabetes insipidus drinking frequently and asking for more cold water
4. Client with hyperthyroidism who has a temperature of 102.2F (39 C) and tachycardia
a. 1, 2, 4, 3
b. 2, 3, 1, 4
c. 3, 4, 2, 1
d. 1, 4, 3, 2 [MAAM MANTE]

33. A client with hypothyroidism is being admitted by the nurse. What type of examination should the nurse conduct to
gain information about this diagnosis?
a. Percuss the thyroid gland
b. Auscultate lung sounds
c. Inspect facial features
d. Inspect ability to ambulate safely
34. The nurse would check for hyperkalemia after unilateral adrenalectomy, which would be suggested by which of the
following?
a. Muscle weakness
b. Constipation
c. Tremors
d. Diaphoresis

35. Which laboratory values would a nurse attending a client with syndrome of inappropriate hormone (SIADH) expect to
find?
a. Serum potassium = 3 mEq/L and high serum osmolality
b. Serum sodium = 150 mEq/L and low urine osmolality
c. Serum sodium = 120 mEq/L and low urine osmolality
d. Serum potassium - 5 mEq/L and low urine osmolality

36. The client with Cushing’s syndrome is given home care instructions by a nurse. The nurse evaluates whether or not
the client comprehends the situation if the client makes which assertion about the hospital discharge instructions?
a. “I need to check the temperature of my legs twice a day”
b. “I need to check the color of my stools”
c. I need to eat foods low in potassium”
d. “I need to take aspirin rather than acetaminophen for a headache”

37. The patient with DI is being cared for by a nurse. Which nursing interventions are the most effective? SELECT ALL
THAT APPLY
1. Monitoring hourly urine output and daily weights
2. Monitoring for signs or symptoms of hyperkalemia
3. Checking urine osmolality and urine ketones
4. Giving (DDAVP) as prescribed
5. Checking glucose levels before meals and at bedtime

38. The nurse is focusing over the serum test report for the adrenocortical insufficiency patient who is in the hospital.
Which value should the nurse inform the Physician about right away?
a. WBC 11,000/mm3
b. Potassium 6.2 mEq/L
c. Glucose 138 mg/dL
d. Sodium 148 mEq/L

39. On days when the client is feeling ill. The clinic nurse teaches a client diagnosed with diabetes mellitus on how to
avoid diabetic ketoacidosis. Which statement by the client indicates the need for further teaching?
a. “If I vomit, I need to stop taking my insulin”
b. “Every 1 to 2 hours, I need to eat 10 to 15 grams of carbohydrates”
c. “If I’m sick for more than 24 hours. I need to call my doctors”
d. “Every 15 to 30 minutes, I need to drink small amounts of fluid”
40. Orders are given to the nurse for the newly admitted Addison’s disease client. Which orders should the nurse ask the
doctor about? SELECT ALL THAT APPLY
1. Potassium 20 m Eq oral now
2. Serum cortisol level in early am
3. 5% dextrose in NS at 100 mL/hr
4. Sodium-restricted diet of 100 mg
5. Obtain serum glucose level now

41. When collecting subjective data from a patient during an evaluation, what should the nurse inquire specifically about
the endocrine system?
a. Energy level [MAAM MANTE]
b. Intake of Vitamin C
c. Frequency of sexual intercourse
d. Employment history

42. Lea is a 35-year-old Hispanic woman. “I’m just not feeling good,” the patient told the doctors. Lea declares that she
has gained a significant amount of weight despite her best efforts to maintain a healthy diet, she just doesn’t seem to be
successful. She has a serum sodium level of 152 mEq/L (152 mmol/L) The normal hormonal response to this situation is
a. Secretion of corticotropin-releasing hormone
b. Secretion of aldosterone
c. Release ACTH
d. Release ADH

43. What physiological response might be expected if the pituitary gland is removed and creates a higher level of ADH?
a. Increase output of urine
b. Decreased output of urine
c. Decreased production of testosterone
d. Increased facial hair growth in women

44. Intracellular receptors are thought to exist in all cells in the body for [INCOMPLETE]
a. Glucagon
b. Growth hormone
c. Thyroid hormone
d. Insulin

45. The nurse is instructing a client who is deficient in parathyroid hormone (PTH) on what foods to eat. Which foods
should be included on a client’s list of appropriate foods?
a. Dark green vegetables, soybeans, and tofu [Q&A. D]
b. Whole grain bread, milk, and liver
c. Spinach, strawberries, and yogurt
d. Rhubarb, yellow vegetales, and fish

46. Blood gas studies should be performed on a patient with DKA because which of the following could occur?
a. Respiratory acidosis
b. Respiratory alkalosis
c. Metabolic alkalosis
d. Metabolic acidosis

47. Hormones are chemical molecules produced by the body that govern and control the activities of specific target cells
or organs. What is the one thing that all hormones have in common?
a. Accelerate the metabolic processes of all body cells
b. Circulate in the blood bound to plasma proteins
c. Enter a cell to alter the cell’s metabolism or gene expression
d. Influence cellular activity of specific target tissues
48. With tachycardia, dyspnea, and occasional chest palpitations, the anxious client is admitted to the hospital.The client’s
blood pressure is 170/110 mm Hg, and his heart rate is 130 beats per minute. Thinning hair, recent 10-pound weight loss,
increased appetite, fine hand and tongue tremors, hyperreflexic tendon reflexes, and smooth, moist skin are all part of the
client’s medical history. Which of the nurse’s advice interventions should be prioritized?
a. Propranolol 2 mg IV q15 min or until symptoms are controlled
b. 12-lead electrocardiogram (ECG) and cardiac enzyme levels
c. Propylthiouracil 600-mg oral loading dose, then 200 mg orally q4h
d. Obtain thyroid-stimulating hormone (TSH) and free T4 levels

49. Four female client’s serum testing results are being reviewed by the nurse. Identify the client who requires the most
immediate attention?
Client Lab Test Results Normal Ranges
A Thyroid-stimulating hormone (TSH) level 5.2 mIU/L 0.4-4.2 mLI/L
Free thyroxine (T4) 0.8ng/dL 0.8-2.7 ng/dL

B Growth Hormone (GH) 23 ug/L 8-18 ug/L


Insulin-like growth factor (IGF-I) 490ng/mL 105-441 ng/mL

C Free Thyroxine (T$) 7.0 ng/dL 0.8-2.7 ng/dL


Thyroid-stimulating hormone (TSH) level 0.1 mIU/L 0.4-4.2 mIU/L

D Fasting Glucose 140 mg/dL 70-110 mg/dL


Hgb a1c 6.9% less than 6%

a. Client B [MAAM MANTE]


b. Client C [correct answer according to Q&A, but wrong ni maam, murag namali siya ani)
c. Client A
d. Client D

50. A patient who has had mitral valve replacement surgery in the past Long-term warfarin (Coumadin) use is diagnosed
with myxedema. When the medication levothyroxine is prescribed, what other types of orders should the nurse anticipate
the doctor to write?
a. Decrease warfarin dosage [MAAM MANTE]
b. Protamine sulfate PRN
c. Weekly APTT measures
d. 2-weeks follow-up with radioactive iodine

51. A patient is undergoing treatment for a hypercalcemic crisis. Which of the assessment results would the nurse
interpret as a sign of overtreatment of the problem?
1. Positive Chvostek sign
2. Leg cramps
3. Bradycardia
4. Decreased bowel sounds
5. Photophobia
6. Vomiting
7. Perioral numbness and tingling

52. To control symptoms, the client with Cushing illness must change his or her diet. Which technique, aside from
boosting protein, would be the most effective?
a. Reduce to 10%
b. Restrict sodium
c. Restrict potassium
d. Increase calories
53. With the client who was newly diagnosed with type 1 diabetes, the nurse intends to discuss diabetic meal planning.
Which of the following actions should the nurse take first?
a. Emphasize the importance of keeping regular meal times every day
b. Encourage use of non-nutritive sweeteners that contain no calories
c. Ask the client to identify favorite foods and the client’s usual mealtimes
d. Teach the client how to count the carbohydrates in meals and snacks

54. A nurse’s abnormal finding during an endocrine assessment would be (select all that apply)
1. Excessive facial hair on a woman
2. Hyperpigmented coloration in lower legs
3. 3-lb weight gain over last 6 months
4. Blood pressure of 100/70 mmHg
5. Soft, formed stool every other day

55. The increased release of corticosteroids is a symptom of Cushing’s disease. The hormones in question are:
a. Catecholamines, glucocorticoids, aldosterone and androgens
b. Adrenocorticotropic hormone (ACTH) [produced by pituitary glands] (depends on context)
c. Glucocorticoids, aldosterone, and androgens [Q&A, but different context] (depends on context)
d. Glucocorticoids and aldosterone
Rationale: Cushing’s is hyperadrenalism which results in elevated corticosteroid, aldosterone, sex hormones and
catecholamines (epinephrine and norepinephrine). ACTH is not produced by the adrenal gland but by the pituitary gland.
Sa Q&A is [the hormones involved are] while sa exam question
is [hormone in question]. Refer to rationale why ACTH is
questioned. Take note nalang sa context of each to answer
correctly.

56. Four clients are being interviewed by the nurse. Which client is most likely to get type 2 diabetes?
a. 40-year-old Asian American female
b. 38-year-old Native American male
c. 25-year-old obese Caucasian male [MAAM MANTE]
d. 56-year-old Hispanic female

57. The nurse is putting together a discharge plan for a postoperative patient who had a heart attack. Adrenalectomy on
one side only. What kind of training should the nurse provide in the strategy for reducing the client’s risk of injury?
a. Explaining the need for lifelong replacement of all adrenal hormones
b. Teaching the client to maintain a diabetic diet
c. Encouraging the adoption of a realistic exercise routine
d. Providing a detailed list of the early signs of a wound infection

58. His thyroidectomy, the nurse instructs his patient on how to support his neck. Which of the following patient
demonstrations would suggest to the nurse that the lesson was effective:
a. The patient covers his ears with both hands and presses firmly
b. The patient grasps his chin with one hand and places the other hand on his forehead
c. The patient places one hand on his forehead and one hand on the back of his head
d. The patient raises his elbows and places his hands behind his neck

59. A continuous intravenous (IV) infusion of insulin is prescribed on a regular basis for a diabetic patient with a blood
glucose level of 700 mg/dL (40 mmol/L). What is the safest way for the nurse to administer this medication?
a. Infuse the medication via an electronic infusion pump
b. Titrate the infusion according to the client’s urine glucose levels
c. Change the solution every 6 hours
d. Mix the solution in 5% dextrose
UNIT EXAM 3 (MEDICAL-SURGICAL 2)

1. Patient De Castro is being prepared to have a craniotomy for a brain tumor. As a patient advocate, you are evaluating
the patient’s understanding of the informed consent before witnessing the patient’s signature on the operative consent
form. Which of the following indicates that the nurse needs to contact the surgeon for further clarification with the
patient?
a. “We talked about the effect of my diabetes on healing”
b. “The surgeon explained how the craniotomy was done"
c. “There are no major risks for the surgery” (Quizlet)
- There are risks with both the surgical procedure and the general anesthesia required for a craniotomy. The risks
involved in the procedure are a part of the informed consent. Other information that is part of an informed consent
includes potential complications, expected benefits, inability of the surgeon to predict results, irreversibility of the
procedure (if applicable), and other available treatments. Talking about the effects of diabetes on healing,
explaining how the craniotomy is performed, and explaining the consequences of declining treatment (e.g., death
if the tumor is not removed) represent appropriate actions to provide information to the client.
d. “I will die if the tumor is not removed from my brain”

2. The earliest sign of hypoxia for an unconscious patient suffered a brain injury:
a. Hypotension
b. Restlessness (Lippincott)
- Restlessness is an early indicator of hypoxia. The nurse should suspect hypoxia in the unconscious client who
becomes restless. The most accurate method for determining the presence of hypoxia is to evaluate the pulse
oximeter value or arterial blood gas values. Cyanosis and decreased respirations are late indicators of hypoxia.
Hypertension, not hypotension, is a sign of hypoxia.
c. Decreased Respiration
d. Cyanosis

3. Which of the following activities would you encourage a client to avoid when there is a risk for increased ICP?
a. Passive ROM
b. Deep Breathing
c. Turning side to sides
d. Coughing (Lippincott)
- Coughing is contraindicated for a client at risk for increased ICP because coughing increases ICP. Deep breathing
can be continued. Turning and passive ROM exercises can be continued with care not to extend or flex the neck.

4. You are assigned to the neurosurgical ward. Your patient was placed on Warfarin therapy, you should monitor for which
of the following laboratory tests?
a. International normalized ratio (Iowa Healthcare)
- To find out how fast your blood clots, a blood test will be performed, specifically the international normalized ratio
(INR) blood test. This is taken from your finger or arm. This blood test is called an International Normalized Ratio
(INR). Your provider may also report your prothrombin time, which is also called protime (PT).
b. Platelet
c. Arterial Blood gas
d. Serum Potassium.

5. You are obtaining a specimen of clear nasal drainage from a patient with head injury. Which of the following tests
differentiates mucus from CSF?
a. Glucose (Quizlet) [Refer to rationale of #11]
b. RBC
c. Microorganism
d. Ph

6. Cushing's triad is a group of symptoms that indicate elevated intracranial


pressure. Which of the following describes the condition's manifestation?
a. Temperature of 98.6 °F
b. Diaphoresis
c. Heart Rate of 48 beats per minute (bradycardia).
d. Biot’s Respiration (an abnormal pattern of breathing characterized by
groups of regular deep inspirations followed by regular or irregular
periods of apnea)
e. Pulse Pressure of 100 mmHg

7. Which manifestation does the nurse assess as a typical reaction to long term phenytoin therapy?
a. Bruises
b. Infection
c. Receding gums (Lippincott)
- A common adverse effect of long-term phenytoin therapy is an overgrowth of gingival tissues. Problems may be
minimized with good oral hygiene, but in some cases, overgrown tissues must be removed surgically. Phenytoin
does not cause weight gain, insomnia, or deteriorating eyesight.
d. Weight Gain
e. Insomnia

8. Following a diagnosis of Multiple Sclerosis, a patient has been discharged from the facilities. The patient will be living
with her 10-year-old grandchild and her 28-year-old daughter. The daughter asks questions as to what she may do at
home to assist her mother. Which of the following measures would be most beneficial to the patient's condition?
a. Assisting the patient on a regular exercise everyday (Lippincott)
- An individualized regular exercise program helps the client to relieve muscle spasms. The client can be trained to
use unaffected muscles to promote coordination because MS is a progressive, debilitating condition. The data do
not indicate that the client needs psychotherapy, day care for the granddaughter, or visits from other clients.
b. Assisting the patient on engaging to psychotherapy
c. Assisting the patient in a scheduled home visit by the rural health nurse
d. Assisting the patient on what foods to eat

9. You are planning the care for a hemiplegic patient to prevent joint deformities of the arm. What position would be
inappropriate?
a. Positioning the hands in a slightly pronated position (Lippincott)
- Placing a pillow in the axilla so the arm is away from the body keeps the arm abducted and prevents skin from
touching skin to avoid skin breakdown. Placing a pillow under the slightly flexed arm so the hand is higher than
the elbow prevents dependent edema. Positioning a hand cone (not a rolled washcloth) in the hand prevents
hand contractures.
- Other contraindicated positions include: immobilizing the extremity in a sling and keeping the arm at the
side using a pillow. Immobilization of the extremity may cause a painful shoulder-hand syndrome. Flexion
contractures of the hand, wrist, and elbow can result from immobility of the weak or paralyzed extremity. It is
better to extend the arms to prevent
b. Positioning a hand cone (roll) in the hand so that the fingers are barely flexed
c. Placing a pillow in the axilla so that the arm is away from the body
d. Placing a pillow under the slightly flexed arm so the hand is higher than the elbow

10. Which of the following nursing actions help a patient with Multiple Sclerosis to avoid episodes of urinary incontinence?
a. Limiting fluid intake to 1 liter a day
b. Inserting an indwelling urinary catheter
c. Administering Glycerin
d. Establishing a regular voiding pattern (Lippincott)
- Maintaining a regular voiding pattern is the most appropriate measure to help the client avoid urinary
incontinence. Fluid intake is not related to incontinence. Incontinence is related to the strength of the detrusor and
urethral sphincter muscles. Inserting an indwelling catheter would be a treatment of last resort because of the
increased risk of infection. If catheterization is required, intermittent self-catheterization is preferred because of its
lower risk of infection. Antibiotics do not infl uence urinary incontinence.

11. Which of the following would the nurse do first when noting a clear drainage on the dressing and bed linen after a
craniotomy?
a. Change the dressing
b. Test the fluid for glucose (Lippincott)
- Glucose in this clear, colorless fluid indicates the presence of cerebrospinal fluid. Excessive fluid leakage should
be reported to the physician. The nurse should not change the dressing of a postoperative craniotomy client
unless instructed to do so by the surgeon. Ordinarily, the head of the bed would not be elevated because this
would put pressure on the sutures. The nurse should notify the physician after testing the fluid for glucose.
c. Elevate the head of the bed
d. Notify the physician

12. The nurse establishes goals for the client nursing care during the acute phase of Guillain Barre Syndrome. The first
priority?
a. Maintaining adequate respiratory function (Lewis)
- Management is aimed at supportive care, particularly ventilatory support, during the acute phase.
b. Reduce Anxiety
c. Maintain mobility of joints in the affected limbs
d. Promote skin Integrity

13. You are caring for a patient for EEG and currently waiting for his breakfast. He is served with soft boiled egg, toast
bread with butter, orange juice and coffee. Which of the following foods should be removed?
a. Soft boiled eggs
b. Orange Juice
c. Coffee (Lippincott)
- Beverages containing caffeine, such as coffee, tea, and cola drinks, are withheld before an EEG because of the
stimulating effects of the caffeine on the brain waves. A meal should not be omitted before an EEG because low
blood sugar could alter brain wave patterns; the client can have the entire meal except for the coffee.
d. Toast bread with butter
14. Which of the following describes decerebrate posturing?
a. Internal rotation and adduction of arms with flexion of elbows, wrists and fingers
b. Back hunched over, rigid flexion of all four extremities with supination of arms and plantar flexion of feet
c. Back arched, rigid extension of all four extremities. (Lippincott)
- Decerebrate posturing occurs in clients with damage to the upper brain stem, midbrain, or pons and is
demonstrated clinically by arching of the back, rigid extension of the extremities, pronation of the arms, and
plantar flexion of the feet. Internal rotation and adduction of arms with flexion of elbows, wrists, and fingers
describes decorticate posturing, which indicates damage to corticospinal tracts and cerebral hemispheres.
d. Supination of arms, dorsiflexion of the feet

15. All except one is an appropriate outcome to establish with a patient who has Multiple Sclerosis?
a. The patient will develop fine mood elevations
b. The patient will develop deeper understanding and ability to comprehend (Quizlet)
c. The patient will develop grasp, lift light weights, sit and stand on a chair
d. The patient will develop ability to walk, row and pulldown

16. Patient G is a 33-year-old female who has had seizures and is at danger of having her ICP raised. What is your
highest priority as the nurse assigned?
a. Decreasing Body core temperature
b. Decreasing systolic pressure
c. Unequal pupil size (Lippincott)
- Increasing ICP causes unequal pupils as a result of pressure on the third cranial nerve. Increasing ICP causes an
increase in the systolic pressure, which refl ects the additional pressure needed to perfuse the brain. It increases
the pressure on the vagus nerve, which produces bradycardia, and it causes an increase in body temperature
from hypothalamic damage
d. Tachycardia

17. Patient L with head injury regains his consciousness after several days being admitted in the hospital. Which of the
following statements made by the nurse is appropriate as patient L awakens?
a. “You are at the hospital patient L. You were in an accident and unconscious for a couple of days”
(Lippincott)
- It is important to first explain where a client is to orient him to time, person, and place. Offering to get his family
and asking him questions to determine whether he is oriented are important, but the first comments should let the
client know where he is and what happened to him. It is useful to be empathetic to the client, but making a
comment such as “I’ll bet you’re a little confused” when he first awakens is not helpful and may cause him anxiety
b. “Hi bet you’re confused right now, am I right?”
c. “I will call your family”
d. “Hello patient L can you tell me where you live?”

18. As a nurse, your expected outcome for a CVA patient placed on thrombolytic therapy?
a. Vasoconstriction
b. Increased vascular permeability
c. Prevention of hemorrhage
d. Dissolved emboli (Lippincott)
- Thrombolytic enzyme agents are used for clients with a thrombotic stroke to dissolve emboli, thus reestablishing
cerebral perfusion. They do not increase vascular permeability, cause vasoconstriction, or prevent further
hemorrhage.

19. When does the nurse encourage a patient with Parkinson’s disease to schedule the most strenuous physical
activities?
a. Early in the morning, when the patient’s energy is very high
b. Immediately after the rest period
c. When the family member will be available
d. It should coincide with the peak action of the drug therapy (Lippincott)
- Demanding physical activity should be performed during the peak action of drug therapy. Clients should be
encouraged to maintain independence in self-care activities to the greatest extent possible. Although some clients
may have more energy in the morning or after rest, tremors are managed with drug therapy.

20. When communicating with a patient who has aphasia. Which of the following nursing actions is inappropriate?
a. Encouraging not to write messages (Lippincott)
- The nurse should encourage the client to write messages or use alternative forms of communication to avoid
frustration. Presenting one thought at a time decreases stimuli that may distract the client, as does speaking in a
normal volume and tone. The nurse should ask the client to “show me” and should encourage the use of gestures
to assist in getting the message across with minimal frustration and exhaustion for the client.
b. Speaking in normal tone
c. Making use of gestures
d. Presenting one thought at a time
21. You are caring for a patient on how to adapt to visual disability. Your primary safety precaution to use is?
a. Turning the head from side to side when walking (Lippincott)
- To expand the visual field, the partially sighted client should be taught to turn the head from side to side when
walking. Neglecting to do so may result in accidents. This technique helps maximize the use of remaining sight.
Covering an eye with a patch will limit the field of vision. Personal items can be placed within sight and reach, but
most accidents occur from tripping over items that cannot be seen. It may help the client to see the door, but
walking presents the primary safety hazard.
b. Wearing an eye patch on the affected eye
c. Placing personal items on the sighted side
d. Lying on bed with the affected side

22. Cerebral Palsy is a term applied to impaired movement resulting from which of the following?
a. Injury to the cerebrum caused by viral infection
b. Nonprogressive Brain damage caused by an injury (Quizlet)
- The term cerebral palsy (CP) refers to a group of nonprogressive disorders of upper motor neuron impairment that
result in motor dysfunction due to injury. In addition, a child may have speech or ocular difficulties, seizures,
hyperactivity, or cognitive impairment. The condition of congenital malformed blood vessels in the ventricles is
known as arteriovenous malformations. Viral infection and metabolic imbalances do not cause CP.
c. Malformed blood vessels in the ventricles caused by genetic
d. Inflammatory brain disease caused by metabolic imbalances

23. During assessment of a patient who sustained a recent spinal cord injury, the nurse auscultates the patient’s
abdomen. The nurse explains to the family that this is necessary because the patient often develops?
a. Profuse Diarrhea
b. Paralytic ileus
- Paralytic Ileus, often associated with spinal shock post an acute spinal cord injury, is an obstruction of the
intestine secondary to paralysis of the intestinal muscles with no evidence of mechanical obstruction, which like
spinal shock can last from a few days to a few weeks.
c. Abdominal Cramping
d. Hyperactive Bowel Sound

24. You are caring for a patient with Multiple Sclerosis and is receiving baclofen. To determine the effectiveness of the
drug, you should assess the?
a. Presence of Muscle spasm
b. Absence of Patellar reflex
c. Absence of involuntary contraction (Lippincott)
- Baclofen is a centrally acting skeletal muscle relaxant that helps relieve the muscle spasms common in MS.
Drowsiness is an adverse effect, and driving should be avoided if the medication produces a sedative effect.
Baclofen does not stimulate the appetite or reduce bacteria in the urine.
d. Presence of increased muscle tone

25. Plasmapheresis achieves this effect by removing which component?


a. Plasma Protein
b. Lymphocytes
c. Antibodies (Google)
- Plasmapheresis is a process that filters the blood and removes harmful antibodies. It is a procedure done similarly
to dialysis; however, it specifically removes antibodies from the plasma portion of the blood.
d. Catecholamines
26. When the patient has a cord transection at T4, which of the following is the primary focus of the nursing assessment?
a. Presence or absence of peripheral pulses (vascular status)
- Although assessment of renal status, gastrointestinal function, and biliary function is important, with the spinal
cord transection at T4 the client's vascular status is the primary focus of the nursing assessment because the
sympathetic feedback system is lost and the client is at risk for hypotension and bradycardia
b. Hyperactive Bowel Sound
c. Urinary Output
d. Liver Function Test

27. Our first priority during the first 24 hours of hospitalization for a comatose patient?
a. Preventing occurrence of impaired skin integrity
- Maintaining intact skin is a priority for the unconscious client. Unconscious clients need to be turned every hour to
prevent complications of immobility, which include pressure ulcers and stasis pneumonia. The unconscious client
cannot be educated at this time. Pain is not a concern. During the first 24 hours, the unconscious client will mostly
likely be on nothing-by-mouth status.
b. Increasing Caloric Intake
c. Initiating a naso gastric tube feeding
d. Minimizing Pain
28. Patient W is a 42-year-old female recently diagnosed with idiopathic Parkinson’s disease. She needs a long time to
complete her morning care, but she becomes annoyed when the nurse in charge offers assistance and refuses all help.
Which statement is the nurse’s best initial response?
a. Suggest to the patient that if she insist on self-care, she should at least modify her routine
b. Tell the patient firmly that she needs assistance and help her with her care
c. Praise the patient for her desire to be independent and give extra encouragement (Lippincott)
- Ongoing self-care is a major focus for clients with Parkinson’s disease. The client should be given additional time
as needed and praised for her efforts to remain independent. Firmly telling the client that she needs assistance
will undermine her self-esteem and defeat her efforts to be independent. Telling the client that her perception is
unrealistic does not foster hope in her ability to care for herself. Suggesting that the client modify her routine
seems to put the hospital or the nurse’s time schedule before the client’s needs. This will only decrease the
client’s self-esteem and her desire to try to continue self-care, which is obviously important to her.
d. Tell the patient that she is being unrealistic about her abilities and must accept the fact that she needs help

29. Which of the following should you observe for a patient in the ictal phase of a grand mal seizure?
a. Body stiffening (T)
b. Violent muscle contraction (T)
c. Facial Grimace
d. Patting Motions
e. Loss of consciousness (T)
Rationale: A generalized tonic-clonic seizure involves both a tonic phase and a clonic phase. The tonic phase consists of
loss of consciousness, dilated pupils, and muscular stiffening or contraction, which lasts about 20 to 30 seconds. The
clonic phase involves repetitive movements. The seizure ends with confusion, drowsiness, and resumption of respiration.
A partial seizure starts in one region of the cortex and may stay focused or spread (e.g., jerking in the extremity spreading
to other areas of the body). An absence seizure usually occurs in children and involves a vacant stare with a brief loss of
consciousness that often goes unnoticed. A complex partial seizure involves facial grimacing with patting and smacking.

30. You are caring for a CVA patient with homonymous hemianopia. You should observe the patient for?
a. Eating food only half of the plate (Lippincott)
- Homonymous hemianopia is blindness in half of the visual field; therefore, the client would see only half of his
plate. Eating only the food on half of the plate results from an inability to coordinate visual images and spatial
relationships. There may be an increased preference for foods high in salt after a stroke, but this would not be
related to homonymous hemianopia. Forgetting the names of foods would be aphasia, which involves a cerebral
cortex lesion. Being unable to swallow liquids is dysphagia, which involves motor pathways of cranial nerves IX
and X, including the lower brainstem.
b. Increased preference for foods high in salt
c. Inability to hold utensils correctly
d. Forgetting the food, the patient is eating

31. A patient with Guillain- Barre syndrome requires a mechanical ventilator. Which of the following is appropriate?
a. Ambulating the patient gradually
b. Maintaining the patient on supine to prevent unnecessary nerve stimulation
c. Turning slowly and gently from side to side (Lippincott)
- Even in the absence of respiratory problems or distress, the child must be turned frequently to help prevent the
cardiopulmonary complications associated with immobility, such as atelectasis and pneumonia. Maintaining the
child in a supine position is unnecessary. Doing so does not prevent unnecessary nerve stimulation. In addition,
maintaining a supine position may lead to stasis of secretions, placing the child at risk for pneumonia. Transferring
the child to a chair will not prevent postural hypotension. However, doing so will increase vascular tone and help
prevent respiratory skin complications. During the acute disease phase, vigorous physiotherapy is contraindicated
because the child may experience muscle pain and be hypersensitive to touch. Careful and gentle handling is
essential.
d. Engaging the patient in a vigorous activity exercises

32. You are caring for a patient with dysphagia about risk for aspiration while eating. Which of the following strategies is
inappropriate?
a. Maintaining an upright position
b. Restricting the diet to liquids until swallowing improves (Lippincott)
- A client with dysphagia (difficulty swallowing) commonly has the most difficulty ingesting thin liquids, which are
easily aspirated. Liquids should be thickened to avoid aspiration. Maintaining an upright position while eating is
appropriate because it minimizes the risk of aspiration. Introducing foods on the unaffected side allows the client
to have better control over the food bolus. The client should concentrate on chewing and swallowing; therefore,
distractions should be avoided.
c. Keeping distraction to a minimum
d. Introducing foods on the unaffected side of the mouth

33. A new medication regimen is being ordered by Doctor Deanne for a patient with Parkinson’s disease. The best time
for the medication to be taken by the patient is?
a. An hour before mealtime
b. Early morning after breakfast
c. At bedtime
d. At the time set by the physician (Lippincott)
- While the client is hospitalized for adjustment of medication, it is essential that the medications be administered
exactly at the scheduled time, for accurate evaluation of effectiveness. For example, levodopa-carbidopa
(Sinemet) is taken in divided doses over the day, not all at one time, for optimum effectiveness.

34. In planning the care for a patient post craniotomy. Which of the following is contraindicated when positioning the
patient?
a. Log rolling
b. Elevating the head using 2-3 pillows (Lippincott)
- Elevating the head of the bed to 30 degrees is contraindicated for infratentorial craniotomies because it could
cause herniation of the brain down onto the brainstem and spinal cord, resulting in sudden death. Elevation of the
head of the bed to 30 degrees with the head turned to the side opposite the incision, if not contraindicated by the
increased intracranial pressure, is used for supratentorial craniotomies.
c. Maintaining the patient flat on bed
d. Keeping the neck in neutral position

35. When administering intermittent enteral feeding for an unconscious patient the nurse must?
a. Weight the patient prior to administering the feeding
b. Determine the exact caloric needs of the patient
c. Obtain a sterile gavage bag and tubing
d. Position the patient on Semi-fowlers position (Lippincott)
- The client should be placed in a semi-Fowler’s position to reduce the risk of aspiration. The formula should be at
room temperature, not heated. Administering enteral tube feedings is a clean procedure, not a sterile one;
therefore, sterile supplies are not required. Clients receiving enteral feedings should be weighed regularly, but not
necessarily before each feeding.

36. Your priority nursing assessment in the first 24 hours after admission of a patient with thrombotic CVA?
a. Pulse Pressure
b. Platelet Count
c. Pupil size and pupillary response (Lippincott)
- It is crucial to monitor the pupil size and pupillary response to indicate changes around the cranial nerves.
d. Neuro Vital signs

37. You are giving mannitol to a patient with increased ICP. You should closely monitor the?
a. Muscle Reflexes
b. Intake and Output (Lippincott)
- After administering mannitol, the nurse closely monitors intake and output because mannitol promotes diuresis
and is given primarily to pull water from the extracellular fluid of the edematous brain. Mannitol can cause
hypokalemia and may lead to muscle contractions, not muscle relaxation. Signs and symptoms, such as widening
pulse pressure and pupil dilation, should not occur because mannitol serves to decrease ICP.
c. Weight
d. Pulse Rate

38. A disorder characterized by an antibody-mediated blockage of neuromuscular transmission, resulting in skeletal


muscle weakening and fast muscle exhaustion:
a. Guillain Barre syndrome
b. Multiple sclerosis
c. Myasthenia gravis (Google)
d. Parkinson’s disease

39. The highest priority for a patient with Guillain Barre syndrome is?
a. Risk for infection due to altered immune system
b. Impaired Swallowing related to neuromuscular impairment
c. Ineffective Breathing pattern related to neuromuscular impairment (ABCD Prioritization)
d. Urinary Incontinence related to muscle paralysis

40. All except one is not a manifestation of Multiple Sclerosis?


a. Diplopia
b. Muscle Tremors
c. Weakness in all extremities
- Guillain-Barre syndrome is a rare disorder in which your body's immune system attacks your nerves. Weakness
and tingling in all extremities are usually the first symptoms. This is not a symptom of multiple sclerosis.
d. Extreme Fatigue

41. You are caring for a patient who has had a head injury and is experiencing clear leakage from the nose. What should
you do first?
a. Give the patient a white pad to collect the fluid (Quizlet) [no rationale]
b. Compress the nares
c. Tilt the head back
d. Administer a post nasal drip

42. An unconscious intubated patient does not have increased ICP. Which nursing actions would be essential?
a. Cleaning the mouth carefully, and applying a thin coat of petroleum jelly on the lips (Lippincott)
- The nurse must clean the unconscious client’s mouth carefully, apply a thin coat of petroleum jelly, and move the
endotracheal tube to the opposite side daily to prevent dryness, crusting, inflammation, and parotiditis. The
unconscious client’s temperature should be monitored by a route other than the oral route (e.g., rectal, tympanic)
because oral temperatures will be inaccurate. The client should be positioned in a lateral or semi prone position,
not a supine position, to allow for drainage of secretions and for the jaw and tongue to fall forward. The client
should not be dragged when turned, as may happen when a drawsheet is used. Care should be taken to lift the
client’s heels, buttocks, arms, and head off of the sheets when turning. Trochanter rolls, splints, foam boot aids,
specialty beds, and so on—not just two pillows—should be used to keep the client in correct body position and to
decrease pressure on bony prominences.
b. Monitoring the oral temperature, keep the room on a room temperature and provide a cooling blanket to the
patient
c. Turning the patient side to side using a draw sheet and placing a pillow behind the back
d. Position the patient on a supine position with head to the side and slightly elevated on 2 pillows

43. You are the general neurology ward's senior nurse. One of the patients exhibits signs of elevated ICP. What are the
first symptoms of deterioration in the patient's condition?
a. Widening of pulse pressure
b. Pulse Rate of 58 Beats per minutes
c. Decreased sensorium (LOC)
- A decrease in the client’s LOC is an early indicator of deterioration of the client’s neurologic status. Changes in
level of consciousness, such as restlessness and irritability, may be subtle. Widening of the pulse pressure,
decrease in the pulse rate, and dilated, fixed pupils occur later if the increased ICP is not treated.
d. Dilated and fixed Pupil

44. You are caring for a patient with a developed autonomic dysreflexia, what should you assess as a possible cause of
this condition?
a. High Grade Fever
b. Decubitus Ulcer (T)
c. Distended bladder (NCBI)
- Autonomic dysreflexia can occur on a daily basis and can be triggered by stimuli such as distension of the bladder
(most common).
d. Phlebitis

45. You observe the patient’s upper arm tremors disappear as the patient unbuttons his Calvin Klein white polo. Which
statement best guides your analysis of these observations about the patient’s Tremors?
a. The tremors disappears when the patient’s attention is diverted by some activity
b. The tremors sometimes disappear with purposeful and voluntary movement (Lippincott)
- Voluntary and purposeful movements often temporarily decrease or stop the tremors associated with Parkinson’s
disease. In some clients, however, tremors may increase with voluntary effort. Tremors associated with
Parkinson’s disease are not psychogenic but are related to an imbalance between dopamine and acetylcholine.
Tremors cannot be reduced by distracting the client.
c. There is no explanation for the observation; it is probably a chance of occurrence
d. The tremors are probably psychological and can be controlled at his will

46. Which of the following nursing measures is inappropriate when providing oral hygiene to a patient who has had CVA?
a. Opening the patient’s mouth with a padded tongue blade
b. Keeping portable suctioning equipment at the bedside
c. Placing the patient on the back with a small pillow under the head (Lippincott)
- A helpless client should be positioned on the side, not on the back, with the head on a small pillow. A lateral
position helps secretions escape from the throat and mouth, minimizing the risk of aspiration. It may be necessary
to suction the client if he aspirates. Suction equipment should be nearby. It is safe to use a padded tongue blade,
and the client should receive oral care, including brushing with a toothbrush.
d. Cleaning the patient’s mouth with teeth with a toothbrush
47. When caring for a client with Myasthenia Gravis, the nurse should assess the client for which of the following
manifestations of Cholinergic Crisis?
a. Decreased secretions and saliva
b. Ptosis (Quizlet) [no rationale]
c. Abdominal cramps (Quizlet) [no rationale]
d. Increased heart rate
e. Fasciculation (Quizlet) [no rationale]
f. Irregular respiratory rate of 6 (Quizlet) [no rationale]

48. A patient with Parkinson’s disease is being placed on levodopa therapy. As a nurse, which of the following indicates
effective therapy?
a. Stable mood
b. Able to comprehend properly
c. Increased on appetite
d. Improvement on muscular rigidity (Quizlet)
- Levodopa is prescribed to decrease severe muscle rigidity. Levodopa does not improve mood, appetite, or
alertness in a client with Parkinson's disease.

49. Which of the following is contraindicated for a patient with seizure precautions?
a. Encouraging the patient to perform his own personal hygiene
b. Instructing patient to be out of bed
c. Assessing the oral temperature using a glass thermometer (Quizlet)
- Temperatures are not assessed orally with a glass thermometer because the thermometer could break and cause
injury if a seizure occurred. The client can perform personal hygiene. There is no clinical reason to discourage the
client from wearing his own clothes. As long as there are no other limitations, the client should be encouraged to
be out of bed.
d. Allowing patient to wear comfortable clothing

50. What nursing assessment should be documented at the beginning of the ictal phase of seizure?
a. Movement of the Head and Eyes (T)
b. Heart Rate and Pulse Rate
c. Type of visual and olfactory aura
d. O2 saturation
e. Muscle rigidity (T)
Rationale: During a seizure, the nurse should note movement of the client’s head and eyes and muscle rigidity, especially
when the seizure first begins, to obtain clues about the location of the trigger focus in the brain. Other important
assessments would include noting the progression and duration of the seizure, respiratory status, loss of consciousness,
pupil size, and incontinence of urine and stool. It is typically not possible to assess the client’s pulse and blood pressure
during a tonic-clonic seizure because the muscle contractions make assessment diffi - cult to impossible. The last dose of
anticonvulsant medication can be evaluated later. The nurse should focus on maintaining an open airway, preventing
injury to the client, and assessing the onset and progression of the seizure to determine the type of brain activity involved.
The type of aura should be assessed in the preictal phase of the seizure.

51. Which neurotransmitter is responsible for many of the functions of the Frontal Lobe
a. Gamma Amino Butyric Acid
b. Dopamine (Google)
- Dopamine, a neurotransmitter that plays a role in reward and motivation, is heavily active in the frontal lobe
because most of the brain's dopamine-sensitive neurons are located here. Attention regulation, including selective
attention.
c. Norepinephrine
d. Histamine

52. Upon awakening from his first tonic-clonic seizure, a 22-year-old patient asked the nurse, “What caused me to have a
seizure? I’ve never had one fore”. Which of the following would the nurse include in the response?
a. Epilepsy
b. Heredity
c. Head trauma
- Trauma is one of the primary causes of brain damage and seizure activity in adults. Other common causes of
seizure activity in adults include neoplasms, withdrawal from drugs and alcohol, and vascular disease. Given the
history of head injury, electrolyte imbalance is not the cause of the seizure. There is no information to indicate that
the seizure is related to a congenital defect. Epilepsy is usually diagnosed in younger clients.
d. Cognitive Deficit
53. The nursing assessment of a patient’s functional status before and after CVA is essential. Why is it so?
a. The rehabilitation plan program will be guided by the it
- The primary reason for the nursing assessment of a client’s functional status before and after a stroke is to guide
the plan. The assessment does not help to predict how far the rehabilitation team can help the client to recover
from the residual effects of the stroke, only what plans can help a client who has moved from one functional level
to another. The nursing assessment of the client’s functional status is not a motivating factor.
b. Functional status before CVA will help predict direct outcomes
c. It will help the patient recognize his physical limitations
d. The patient can be expected to regain much of his functioning

54. You are caring for a seizure patient who is on Dilantin. What is your best response to the patient's question about how
the drug is helping his condition?
a. Transmission of abnormal impulses in the spinal cord is depressed
b. The responsiveness of neurons in the brain to abnormal impulses is reduced (Lippincott)
- The most common theory is that it reduces the responsiveness of neurons in the brain to abnormal impulses-that
is, it depresses neural activity. Dilantin does not influence norepinephrine or transmission of impulses in the spinal
cord, nor does it interrupt the flow of abnormal impulses from peripheral neurons in the viscera to the brain.
c. It corrects the abnormal synthesis of norepinephrine in the body
d. It interrupts the flow of abnormal impulses from the peripheral neurons in the viscera to the brain

55. Patient H arrives in the ED with an ischemic CVA and receives tissue plasminogen activator administration. Which
nursing assessment is your priority?
a. Current Medications
b. Complete nursing and physical history
c. Time of onset of CVA (Lippincott)
- Studies show that clients who receive recombinant t-PA treatment within 3 hours after the onset of a stroke have
better outcomes. The time from the onset of a stroke to t-PA treatment is critical. A complete health assessment
and history is not possible when a client is receiving emergency care. Upcoming surgical procedures may need to
be delayed because of the administration of t-PA, which is a priority in the immediate treatment of the current
stroke. While the nurse should identify which medications the client is taking, it is more important to know the time
of the onset of the stroke to determine the course of action for administering t-PA.
d. Complete blood count

56. A patient is experiencing expressive aphasia, which nursing interventions is your priority in terms of communication?
a. Speaking loudly
b. Using a picture board (Lippincott)
- Expressive aphasia is a condition in which the client understands what is heard or written but cannot say what he
or she wants to say. A communication or picture board helps the client communicate with others in that the client
can point to objects or activities that he or she desires.
c. Writing directions so that patient can read them
d. Speaking in short sentences

57. You are assessing the level of consciousness in a client with head injury who has been unresponsive for the last 8
hours. Using the GCS you notice that the patient opens the eyes only as a response to pain, responds without sounds
that are not understandable, and has an abnormal extension of the extremities. You should:
a. Document the level of consciousness as coma (Quizlet)
- The client has a score of 6 (eye opening to pain=2; verbal response, incomprehensible sounds=2, best motor
response, abnormal extension=2); a score >7 is indicative of coma. While the nurse should continue to speak to
the client, at this time the client will not be able to be aroused. The nurse should continue to provide skin care and
appropriate alignment, but the client will continue to have a motor response of limb extension. It is not necessary
to notify the HCP as this assessment does not represent a significant change in neurological status.
b. Attempt to arouse the client
c. Reposition the client with extremities in normal alignment
d. Notify the physician

58. Your patient is being switched from levodopa to carbidopa. You should monitor for which of the following possible
complications during medication shifting and dosage titration?
a. Fatigue, Abdominal pain, Weight loss and vomiting
b. Dry skin, Constipation, descending weak muscles.
c. Disorientation, Hallucinations, Mood swings and Paranoia.
d. Blood Pressure, Pulse Rate, Respiratory Rate, Temperature and O2 saturation alteration (Vital signs)
(Lippincott)
- Vital signs should be monitored, especially during periods of adjustment. Changes, such as orthostatic
hypotension, cardiac irregularities, palpitations, and light-headedness, should be reported immediately.

59. Which of the following techniques does the nurse avoid when changing a patient’s position in bed if the patient has
hemiparesis?
a. Rolling the patient onto his side
b. Lifting the patient when moving him up to bed
c. Having the patient help lift herself off the bed using a trapeze
d. Sliding the patient to move up in bed (Lippincott)
- Sliding a client on a sheet causes friction and is to be avoided. Friction injures skin and predisposes to pressure
ulcer formation. Rolling the client is an acceptable method to use when changing positions as long as the client is
maintained in anatomically neutral positions and her limbs are properly supported. The client may be lifted as long
as the nurse has assistance and uses proper body mechanics to avoid injury to himself or herself or the client.
Having the client help lift herself off the bed with a trapeze is an acceptable means to move a client without
causing friction burns or skin breakdown.

60. You are caring for patient D recently diagnosed with Multiple Sclerosis, who is manifesting slurred speech, which of
the following nursing actions is contraindicated?
a. Instructing the patient to speak clearly
b. Asking the patient to repeat indistinguishable words
c. Encouraging the patient to speak slowly
d. Advising the patient to speak louder (Lippincott)
- Asking a client to speak louder even when tired may aggravate the problem. Asking the client to speak slowly and
distinctly and to repeat hard-to understand words helps the client to communicate effectively.

61. What information about Bell’s palsy is TRUE? [ISSUE]


a. Herpes simplex virus 1 is strongly associated as a precipitating factor in the development of Bell's palsy. [TRUE]
b. Moist heat, gentle massage, electrical stimulation of the nerve, and exercises are prescribed to treat Bell's palsy.
[TRUE]
c. An inability to close the eyelid, with an upward movement of the eyeball when closure is attempted, is evident.
[TRUE]
d. Nutrition and avoidance of hot foods or beverages are special needs of this patient. [NOT TRUE]

INSTEAD: A patient is diagnosed with Bell's palsy. What information should the nurse teach the patient about Bell's
palsy (select all that apply)?
a. Bell's palsy affects the motor branches of the facial nerve.
b. Antiseizure drugs are the drugs of choice for treatment of Bell's palsy.
c. Nutrition and avoidance of hot foods or beverages are special needs of this patient.
d. Herpes simplex virus 1 is strongly associated as a precipitating factor in the development of Bell's palsy.
e. Moist heat, gentle massage, electrical stimulation of the nerve, and exercises are prescribed to treat Bell's
palsy.
f. An inability to close the eyelid, with an upward movement of the eyeball when closure is attempted, is
evident.
Rationale: Bell's palsy affects the motor branches of the facial nerve. It is treated with corticosteroids, usually prednisone.
Herpes simplex virus 1 may be a precipitating factor. Moist heat, gentle massage, electrical nerve stimulation, and
exercises are prescribed. Care must be taken to protect the eye with sunglasses, artificial tears or gel, and possibly taping
the eyelid closed at night. Oral hygiene is important but avoidance of hot foods is not needed.

62. You are caring for a Parkinson’s disease patient who has recently been diagnosed. Which of the following teaching
plans should be discussed the most?
a. Maintaining a balanced nutrition
b. Enhancing the immune system free from any infections
c. Engaging the patient in different diversional activities
d. Maintaining a safe environment to the patient (Lippincott)
- The primary focus is on maintaining a safe environment because the client with Parkinson’s disease usually has a
propulsive gait, characterized by a tendency to take increasingly quicker steps while walking. This type of gait
commonly causes the client to fall or to have trouble stopping. The client should maintain a balanced diet,
enhance the immune system, and enjoy diversional activities; however, safety is the primary concern

63. Which nursing interventions are most effective in preventing plantar flexion in a patient who has had CVA with
paralysis noted?
a. Reposition the patient every 2 hours
b. Have the patient wear ankle high fitted shoes at interval throughout the day (Lippincott)
- The use of ankle-high tennis shoes has been found to be most effective in preventing plantar flexion (footdrop)
because they add support to the foot and keep it in the correct anatomic position. Footboards stimulate spasms
and are not routinely recommended. Regular repositioning and range-of-motion exercises are important
interventions, but the client’s foot needs to be in the correct anatomic position to prevent overextension of the
muscle and tendon. Massaging does not prevent plantar flexion and, if rigorous, could release emboli
c. Place the patient feel against a firm footboard
d. Massage the patient’s feet and ankle regularly
64. Which of the following respiratory patterns indicate increasing ICP?
a. Nasal flaring
b. Irregular respiration (Lippincott)
c. Rapid respiration
d. Shallow respiration
e. Slow respiration (Lippincott)

ALTERNATIVE:
a. Slow, irregular respirations (Lippincott)
- Neural control of respiration takes place in the brain stem. Deterioration and pressure produce irregular
respiratory patterns. Rapid, shallow res- pirations, asymmetric chest movements, and nasal flaring are more
characteristic of respiratory distress or hypoxia.
b. Rapid, shallow respirations.
c. Asymmetric chest excursion.
d. Nasal flaring.

65. After the patient sustained cord injury 3 months ago, the patient complains of a pounding headache. The nurse notes
that the patient’s arm and face are flushed and he is also diaphoretic. Which action should the nurse do next?
a. Positioning patient flat on bed
b. Checking the patency of the urinary catheter (Quizlet) [no rationale]
c. Preparing to administer epinephrine
d. Lowering the patient’s head below his knees

66. Which manifestation does the nurse expect in the patient in the post-ictal phase of grand mal seizure?
a. Drowsiness (Lippincott)
- The nurse should expect a client in the postictal phase to experience drowsiness to somnolence because
exhaustion results from the abnormal spontaneous neuron fi ring and tonic-clonic motor response. An inability to
move a muscle part is not expected after a tonic-clonic seizure because a lack of motor function would be related
to a complication, such as a lesion, tumor, or stroke, in the correlating brain tissue. A change in sensation would
not be expected because this would indicate a complication such as an injury to the peripheral nerve pathway to
the corresponding part from the central nervous system. Hypotension is not typically a problem after a seizure.
b. Hypotension
c. Inability to move
d. Paresthesia

67. You are the nurse assigned to a Parkinson’s disease patient. Which of the following are the first symptoms of the
condition?
a. Bradykinesia
b. Akinesia
c. Unintentional tremors (tremors)
d. Muscular rigidity (rigor)

68. The priority nursing intervention in the post-ictal phase of seizure?


a. Reorient the patient to the time, place and person
b. Determine the patient’s level of sleepiness
c. Position the patient comfortably
d. Assess the breathing pattern (Lippincott)
- A priority for the client in the postictal phase (after a seizure) is to assess the client’s breathing pattern for effective
rate, rhythm, and depth. The nurse should apply oxygen and ventilation to the client as appropriate. Other
interventions, to be completed after the airway has been established, include reorientation of the client to time,
person, and place. Determining the client’s level of sleepiness is useful, but it is not a priority. Positioning the
client comfortably promotes rest but is of less importance than ascertaining that the airway is patent.

69. You are caring for a Parkinson’s disease patient who was recently diagnosed. Which of the following teaching plans
should receive the greatest attention?
a. Beginning to prepare for terminal case
b. Stopping the progression of the disease (T)
c. Maintaining the optimal body function (Lippincott)
- Helping the client function at his or her best is most appropriate and realistic. There is no known cure for
Parkinson’s disease. Parkinson’s disease progresses in severity, and there is no known way to stop its
progression. Many clients live for years with the disease, however, and it would not be appropriate to start
planning terminal care at this time.
d. Treating the manifestations

70. Patient Y undergoes a craniotomy for an infratentorial brain tumor. The nurses place the patient in which position?
a. Trendelenburg
b. Prone
c. Semi-fowler’s
d. Side-lying

71. The patient is to have CT Scan without contrast in the morning. Which statement by the nurse would be appropriate?
a. You must shampoo your hair tonight to remove all the dirt and oil
b. You will have some hair shaved to attach the small electrodes to your scalp
c. You will need to hold your head still during the examination (Lippincott)
- The client will be asked to hold the head very still during the examination, which lasts about 30 to 60 minutes. In
some instances, food and fl uids may be withheld for 4 to 6 hours before the procedure if a contrast medium is
used because the radiopaque substance sometimes causes nausea. There is no special preparation for a CT
scan, so a shampoo the night before is not required. The client may drink fl uids until 4 hours before the scan is
scheduled. Electrodes are not used for a CT scan, nor is the head shaved.
d. You may drink fluids until early morning but after that drink nothing until the scan is completed

72. The major purpose of coordinating with a physical therapist for a patient with Parkinson’s disease is to?
a. To reduce ataxia
b. To maintain joint flexibility (Lippincott)
- The client will be asked to hold the head very still during the examination, which lasts about 30 to 60 minutes. In
some instances, food and fluids may be withheld for 4 to 6 hours before the procedure if a contrast medium is
used because the radiopaque substance sometimes causes nausea. There is no special preparation for a CT
scan, so a shampoo the night before is not required. The client may drink fluids until 4 hours before the scan is
scheduled. Electrodes are not used for a CT scan, nor is the head shaved.
c. To build muscle strength
d. To improve muscle endurance

73. You are caring for an unconscious patient, your highest nursing action to maintain cerebral perfusion is?
a. Using a French 14 when suctioning
b. Allowing the patient to have adequate rest period
c. Hyperoxygenate the patient before and after suctioning (Quizlet) [no rationale]
d. Suctioning the patient occasionally

74. A patient tells you that he is unclear about what an aura is. Your best response to the patient’s concern is?
a. A feeling of relaxation as the seizure begins to subside
b. A symptom that occurs just before seizure attack (Lippincott)
- An aura is a premonition of an impending seizure. Auras usually are of a sensory nature (e.g., an olfactory, visual,
gustatory, or auditory sensation); some may be of a psychic nature. Evaluating an aura may help identify the area
of the brain from which the seizure originates. Auras occur before a seizure, not during or after (postictal). They
are not similar to hallucinations or amnesia or related to relaxation.
c. A post-ictal state of amnesia
d. A hallucination that occurs during seizure

75. An unconscious patient with multiple head injuries arrives in the emergency department. As a nurse, your utmost
priority is?
a. Administering fresh whole blood
b. Putting a pressure pad at the bleeding site
c. Giving supplemental oxygen (Lippincott)
- The highest priority for a client with multiple injuries is to establish an open airway for effective ventilation and
oxygenation. Unless the client has a patent airway, other care measures will be futile. Replacing blood loss,
stopping bleeding from open wounds, and checking for a neck fracture are important nursing interventions to be
completed after the airway and ventilation are established.
d. Assessing for neck fracture

76. The nurse has established a goal to maintain increased ICP within the normal range for a client who had a craniotomy
12 hours ago. What should the nurse do?
a. Encourage the client to cough to expectorate secretions
b. Elevate the head of the bed to 15-30 degrees (Lippincott)
c. Stimulate the client with active range of motion exercises
d. Monitor neurologic status using Glasgow Coma Scale (Lippincott)
e. Notify the doctor if the ICP is greater than 20mmHg (Lippincott)
Rationale: The nurse should maintain ICP by elevating the head of the bed and monitoring neurologic status. An ICP
greater than 20 mm Hg indicates increased ICP, and the nurse should notify the health care provider. Coughing and
range-of-motion exercises will increase ICP and should be avoided in the early postoperative stage.

77. As a registered nurse assigned to the neurology department. Which of the following interventions is appropriate for a
patient with an ICP of 20 mmHg?
a. Restricting fluids
b. Giving a warm blanket
c. Encourage the patient to hyperventilate (Lippincott)
- Normal ICP is 15 mm Hg or less for 15 to 30 seconds or longer. Hyperventilation causes vasoconstriction, which
reduces cerebrospinal fluid and blood volume, two important factors for reducing a sustained ICP of 20 mm Hg. A
cooling blanket is used to control the elevation of temperature because a fever increases the metabolic rate,
which in turn increases ICP. High doses of barbiturates may be used to reduce the increased cellular metabolic
demands. Fluid volume and inotropic drugs are used to maintain cerebral perfusion by supporting the cardiac
output and keeping the cerebral perfusion pressure greater than 80 mm Hg
d. Administering low dose of barbiturates

78. Which of the following interventions is most effective in minimizing the risk of seizure activity in a patient who is
undergoing diagnostic evaluation after having experienced several episodes of seizures?
a. Administer Carbamazepine 200MG orally (Lippincott)
- Carbamazepine (Tegretol) is an anticonvulsant that helps prevent further seizures. Bed rest, sedation
(phenobarbital), and providing privacy do not minimize the risk of seizures.
b. Administer Phenobarbital 30 MG orally
c. Close the door to the room to minimize stimulation
d. Maintain the patient on bed rest

79. During the first 24 hours after thrombolytic treatment for an ischemic CVA, the primary goal is to control the patients?
a. Temperature
b. Blood pressure (Lippincott)
- Control of blood pressure is critical during the first 24 hours after treatment because an intracerebral hemorrhage
is the major adverse effect of thrombolytic therapy. Vital signs are monitored, and blood pressure is maintained as
identified by the physician and specific to the client’s ischemic tissue needs and risk of bleeding from treatment.
The other vital signs are important, but the priority is to monitor blood pressure
c. Pulse
d. Respiration

Unit Exam 4 (Medical-Surgical Nursing 2)

1. A client learns that his left ankle is sprained. He asked the nurse, “What is a sprain?”. Which of the following replies is
most appropriate?
a. a sprain is an overstretching of the joint muscles.
b. a sprain is an injury to the ligaments of the joint caused by overuse or twisting motion.
c. a sprain is an injury to soft tissue & is minor compared to a fracture.
d. a sprain is an injury to the tendons & muscles caused by rotational pull.

2. A patient with a comminuted fracture of the right femur has Buck's traction in place while waiting for surgery. To assess
for pressure areas on the patient's back and sacral area and to provide skin care, the nurse should
a. have the patient lift the buttocks by bending and pushing with the left leg.
b. place a pillow between the patient's legs and turn gently to each side.
c. loosen the traction and have the patient turn onto the unaffected side.
d. turn the patient partially to each side with the assistance of another nurse.

3. Following x-rays of an injured wrist, the patient is informed that it is badly sprained. In teaching the patient to care for
the injury, the nurse tells the patient to (PRICE: protect, rest, ice, compression, elevation)
a. apply a heating pad to reduce muscle spasms.
b. use pillows to keep the arm elevated above the heart. (Ma’am E)
c. wear an elastic compression bandage continuously.
d. gently exercise the joint to prevent muscle shortening.

4. The nurse has notes that the physician has a diagnosis of presbycusis on the client's chart. The nurse plans care
knowing the condition is:
a. A conductive hearing loss that occurs with aging.
b. sensorineural hearing loss that occurs with aging
c. Nystagmus that occurs with aging
d. Tinnitus that occurs with aging

5. A patient sustained a fracture to the femur. The patient has suddenly become confused, restless, and has a respiratory
rate of 30 breaths per minute. Based on the location of fracture and the presenting symptoms, this patient may be
experiencing what type of complication?
a. Compartment Syndrome
b. Hypovolemia
c. Fat embolism
d. Osteomyelitis

6. Osteomyelitis is an infection of the bone most commonly caused by:


a. Staphylococcus Aureus
b. Pseudomonas
c. Mycobacterium
d. Streptococcus

7. Which of the following children is at risk of recurrent otitis media (OM)?


a. An 18-month-old infant who lives with a smoker
b. A 2-year-old child who has had two ear infections in the past 6 months
c. An 18-month-old infant who has had three episodes of ear infections in a 5-month period
d. A 6-month-old infant who has a sibling who had tubes inserted at 3 years of age

8. A tough protein & deposits of many minerals which gives tensile strength to the bones is called:
a. matrix
b. bursae
c. Collagen
d. cartilage

9. The nurse is developing a teaching plan for the client with glaucoma. Which of the following instructions would the
nurse include in the plan of care?
a. Avoid overuse of the eyes
b. Decrease fluid intake to control the intraocular pressure
c. Eye medications will need to be administered lifelong.
d. Decrease the amount of salt in the diet

10. A client is diagnosed with a disorder involving the inner ear. Which of the following is the most common client
complaint associated with a disorder in this part of the ear?
a. Burning of the ear
b. Hearing loss
c. Pruritus
d. Tinnitus

11. A patient is seen at the urgent care center after falling on the right arm and shoulder. It will be most important for the
nurse to determine:
a. The amount of pain the patient is experiencing
b. Whether the right arm is shorter than the left
c. Whether there is bruising at the shoulder area
d. How much range of motion (ROM) is present

12. A nurse would question an order to irrigate the ear canal in which of the following circumstances?
a. Ear pain
b. Perforated tympanic membrane
c. Hearing loss
d. Otitis externa

13. The nurse is performing a voice test to assess hearing. Which of the following describes the accurate procedure for
performing this test?
a. Whisper a statement while the client blocks both ears.
b. Whisper a statement and ask the client to repeat it (mmaamm E)
c. Stand 4 feet away from the client to ensure that the client can hear at this distance.
d. Whisper a statement with the examiners back facing the client (Quizlet)

14. Cataract surgery results in aphakia. Which of the following statements best describes this term?
a. A “keyhole” pupil
b. Loss of accommodation
c. Absence of the crystalline lens
d. Retinal detachment

15. A client has been diagnosed with OA of the left hip. Which of the following factors would most likely increased the joint
symptoms of OA?
a. Obesity
b. a long history of smoking
c. emotional stress
d. excessive alcohol use

16. When working with a patient whose job involves many hours of word processing, the nurse will teach the patient about
the need to:
a. Obtain a keyboard pad to support the wrist while word processing.
b. Use acetaminophen (Tylenol) instead of NSAIDs for wrist pain.
c. Wrap the wrists with a compression bandage every morning.
d. Do stretching and warm-up exercises before starting work.

17. The client diagnosed with Rheumatoid Arthritis has been taking large doses of aspirin to relieve her joint pain. The
nurse should assess the client for w/c important symptom of aspirin toxicity?
a. Tinnitus
b. drowsiness
c. dysuria
d. chest pain

18. The nurse is performing an admission assessment on a client with a diagnosis of detached retina. Which of the
following is associated with this eye disorder?
a. Pain in the affected eyel
b. Total loss of vision
c. A yellow discoloration of the sclera.
d. A sense of a curtain falling across the field of vision

19. A child is in Bryant traction due to fractured left femur. The nurse should make sure that he is positioned on his back
with his legs
a. flexed at 30 degree angle at the hips, with the hips touching the bed
b. flexed at a 90 degree angle, with the hips slightly off the bed
c. straight in line, with his body resting flat on bed
d. separated, with the affected leg at a right angle to the body & the unaffected leg in any comfortable position

20. For a client having an episode of acute narrow-angle glaucoma. a nurse expects to give which of the following
medications?
a. Furosemide (Lasix)
b. Acetazolamide (Diamox)
c. Urokinase (Abbokinase)
d. Atropine
21. The nurse is caring for a client who is scheduled for an MRI study. Which statement made by the client warrants
further assessment by the nurse?
A. “My doctor told me never to take laxatives.”
B. “I had a total hip replacement 5 years ago.”
C. “I am allergic to iodine & seafood.”
D. “I had a total hip replacement 5 years ago.”

22. The nurse has conducted discharge teaching for a client who had a fenestration procedure for the treatment of
otosclerosis. Which of the following, If stated by the client would indicate that teaching was effective?
A. “I will take stool softeners as prescribed by my doctor.”
B. “It’s ok to take a shower and wash my hair.”
C. “I should drink liquids through a straw for the next 2-3 weeks.”
D. “I can resume my tennis lessons starting next week.”

23. A patient arrives in the emergency department with ankle swelling and severe pain after twisting the ankle playing
soccer. All of the following orders are written by the health care provider. Which one will the nurse act on first?
A. Give acetaminophen with codeine (Tylenol #3).
B. Administer naproxen (Naprosyn) 500 mg PO.
C. Wrap the ankle and apply an ice pack.
D. Take the patient to the radiology department for x-rays.

24. A patient who had undergone ORIF asked the nurse what are the functions of the bones. Which of the following does
not fit the description?
A. assist in regulating calcium and phosphate concentration
B. support & protects structures of the body
C. provide attachment for muscles, tendons and ligaments
D. ends of long bones aid in the formation of blood cells

25. When counseling an older patient about ways to prevent fractures, which information will the nurse include?
A. Tacking down scatter rugs in the home is recommended.
B. Buying shoes that provide good support and are comfortable to wear is recommended
C. Most falls happen outside the home.
D. Occasional weight-bearing exercise will improve muscle and bone strength.

26. The nurse is caring for a client with a diagnosis of detached retina. Which assessment sign would indicate that
bleeding has occurred as a result of the retinal detachment?
A. A sudden sharp pain in the eye
B. Total loss of vision
C. Complaints of a burst of black spots or floaters
D. A reddened conjunctiva

27. The client with glaucoma asks the nurse is complete vision will return. The most appropriate response is:
A. "Your vision loss is temporary and will return in about 3-4 weeks."
B. Although some vision has been lost and cannot be restored, further loss may be prevented by adhering
to the treatment plan."
C. "Your vision will return as soon as the medications begin to work."
D. "Your vision will never return to normal."

28. Following an Above Knee Amputation of the leg, a client complains of pain in the foot that is no longer there. The
nurse understands that the patient is experiencing:
A. phantom limb pain (Ma’am E)
B. phantom limb sensation
C. denial
D. Hallucination

29. The nurse is caring for a client that is hearing impaired. Which of the following approaches will facilitate
communication?
A. Speak frequently
B. Speak in a normal tone
C. Speak directly into the impaired ear
D. Speak loudly

30. A patient with traction for his fracture must be aware on how to take care of it. Which of the ff is not correct?
A. maintain proper body alignment
B. ensure that the weights hang freely & do not touch the floor
C. remove the weight temporarily when changing the bed linens
D. ensure that pulleys are not obstructed

31. The nurse understands that the joint most likely to be involved in a client with osteoarthritis are the:
A. cervical spine & shoulders
B. hips & knees
C. fingers & metacarpals
D. ankles & metatarsals

32. When a client has paraplegia, the least effective method of preventing contractures of the joints of the lower
extremities would be:
A. maintaining proper bed positions
B. providing the client with active exercise instructions
C. passively moving the extremities through ROM several times daily
D. changing bed position every 2hrs

33. The client is being discharged from the ambulatory care unit following cataract removal. The nurse provides
instructions regarding home care. Which of the following, if stated by the client, indicates an understanding of the
instructions?
A. "I will wear my eye shield at night and my glasses during the day."
B. "I will take Aspirin if I have any discomfort."
C. "I will sleep on the side that I was operated on."
D. "I will not lift anything if it weighs more that 10 pounds."

34. The synovial fluid of the joints minimizes?


A. work output
B. friction in the joints
C. velocity of movements
D. Efficiency

35. This complication is due to increased pressure within 1 or more compartment causing massive compromised
circulation to an area and will lead to neuromuscular damage.
A. avascular necrosis
B. fat embolism
C. compartment syndrome
D. pulmonary embolism

36. Which of the following procedures or assessments must the nurse perform when preparing a client for eye surgery?
A. Clipping the client’s eyelashes
B. Verifying the client has been NPO since midnight. or at least 8 hours before surgery.
C. Verifying the affected eye has been patched 24 hours before surgery
D. Obtaining informed consent with the client’s signature and placing the forms on the chart.

37. A patient diagnosed with RA was admitted due to impaired physical immobility. The client asked the nurse why her
joints are increasingly painful. The nurse’s response would be based on the knowledge that RA:
A. is usually caused by aging
B. begins with inflammation of joints synovial tissue
C. affects only the weight bearing joints of the body
D. results from degenerative joint damage

38. A checkout clerk in a grocery store has muscle and tendons that have become inflamed, causing pain and weakness
in the left hand and elbow. The nurse identifies these symptoms as related to:
A. Meniscus injury
B. Muscle spasms
C. Carpal tunnel syndrome
D. Repetitive strain injury

39. A patient has been diagnosed to have an open fracture. This is considered a medical emergency because of:
A. severe muscle spasm & pain
B. poor bone healing
C. high risk for deformity
D. high risk for infection (Ma’am E)

40. Spasmodic torticollis is an example of:


A. Sprain
B. Subluxation
C. Contusion
D. Strain

41. The client arrives in the emergency room after sustaining a chemical eye injury from a splash of battery acid. The
initial nursing action is to:
A. Swab the eye with antibiotic ointment
B. Irrigate the eye with sterile normal saline
C. Cover the eye with a pressure patch.
D. Begin visual acuity testing

42. A nurse is caring for a toddler who has acute otitis media. Which of the following is the priority action for the nurse to
take?
A. Administer analgesics
B. Educate the family on care of the child.
C. Prevent clinical complications.
D. Provide emotional support to the family.

43. A male patient was brought to the ER because of a vehicular accident. He sustained multiple fractures on his right
femur with more than 3 fragments noted in the xray. This type of fracture is called?
A. Comminuted
B. impacted
C. greenstick
D. Transverse

44. A client arrives at the emergency room with a foreign body in the left ear that has been determined to be an insect.
Which intervention would the nurse anticipate to be prescribed initially?
A. Instillation of antibiotic ear drops
B. Instillation of corticosteroids ear drops
C. Irrigation of the ear
D. Instillation of diluted alcohol

45. A nurse is caring for a 2-year-old child who has had three ear infections in the past 5 months. The nurse should know
that the child is at risk for developing which of the following as a long-term complication?
A. Balance difficulties
B. Prolonged hearing loss
C. Speech delays
D. Mastoiditis

46. Rehabilitation of a client scheduled for an amputation should begin:


A. when its time for prosthesis
B. during the convalescent stage
C. before the surgery
D. on discharge from the hospital

47. During the early postoperative period. The client who had a cataract extraction complains of nausea and severe eye
pain over the operative site. The initial nursing action is to:
A. Reassure the client that this is normal.
B. Call the physician
C. Administer the ordered main medication and antiemetic
D. Turn the client on his or her operative side

48. Which of the following would the nurse instruct the client with gouty arthritis to avoid when starting a low purine diet?
A. citrus fruits
B. organ meats
C. fresh fish
D. green vegetables

49. A 6 year old child broke his arms after falling to the ground while riding a bike. What type of fracture is most commonly
found in children?
A. Greenstick
B. impacted
C. depressed
D. comminuted

50. Which assessment finding found while assessing a patient with a fracture who has traction requires immediate
intervention?
A. Pin sites are free from drainage.
B. The weights are freely hanging on the floor
C. Patient uses the overhead trapeze bar to move around in the bed.
D. Patient’s extremities have a capillary refill of less than 2 seconds.
51. To prevent infection and osteomyelitis, the nurse provides pin site care and inspects the site daily for evidence of
infection. Which of the ff clinical manifestations at the pin site would alert the nurse to infection?
Select the correct response:
A. lack of scab formation
B. slight serous oozing
C. Pain
D. itching

52. Which of the following symptoms would occur in a client with a detached retina?
A. Flashing lights and floaters
B. Homonymous hemianopia
C. Loss of central vision
D. Ptosis

53. When developing a teaching session on glaucoma for the community. Which of the following statements would the
nurse stress?
A. Yearly screening for people ages 20-40 years is recommended.
B. Glaucoma is easily corrected with eyeglasses
C. Glaucoma can be painless and vision may be lost before the person is aware of a problem (Ma’am E)
D. White and Asian individuals are at the highest risk for glaucoma.

54. Which of the following behaviors demonstrated by an individual may be indicative of hearing loss?
A. Speaking slowly with well-articulated consonants
B. Not looking at the examiner when being questioned
C. Frequently asking for the question to be repeated
D. Talking in a high-pitched voice

55. A male client has just had a cataract operation without a lens implant. In discharge teaching. the nurse will instruct the
client’s wife to:
A. Have her husband remain in bed for 3 days
B. Keep the eye dressing on for one week
C. Feed him soft foods for several days to prevent facial movement
D. Allow him to walk upstairs only with assistance.

56. Flexion of the forearm is an example of what type of muscle contraction?


A. Isotonic
B. isometric
C. combined

57. A client with Meniere's disease is experiencing severe vertigo. Which instruction would the nurse give to the client to
assist in controlling the vertigo?
A. Avoid sudden head movements (Ma’am E)
B. Increase sodium in the diet
C. Lie still and watch the television
D. Increase fluid intake to 3000 ml a day

58. A patient is scheduled for closed reduction of a fractured radius. This procedure:
A. is performed under topical anesthesia
B. needs application of a cast
C. needs general anesthesia
D. involves manual manipulation of the affected part (Ma’am E)

59. A patient was brought to the ER due to fall. Upon physical examination, the nurse noticed that the patient’s right leg is
shorter & twisted from its normal position. The xray showed femoral head dislocation. This type of joint is described as:
A. Synarthrosis
B. Ampiarthrosis
C. Condyloid
D. Diarthrosis

60. A 5 year old has a fracture of the right upper arm. The x-ray showed that one side of the bone is bent while the other
is broken. This known as a __________ fracture?
A. Spiral
B. Greenstick
C. Oblique
D. Transverse

61. A patient with a fractured radius asks when the cast can be removed. The nurse will instruct the patient that the cast
can be removed only after the bone
A. is strong enough to stand mild stress.
B. union is complete on the x-ray.
C. fragments are fully fused.
D. healing has started.

62. A 22-year-old patient started an exercise regimen 2 months ago that includes running 3 to 4 miles a day. The patient
tells the nurse, "I enjoy my daily runs, but now I have shin splints." Which response by the nurse is appropriate?
A. "You should expect some leg pain while running."
B. "You should try speed-walking rather than running."
C. "You may be increasing your running time too quickly and need to cut back a little bit."
D. "You need to have x-rays of your lower legs to be sure you do not have stress fractures."

63. What is a late sign of compartment syndrome? [5PS: Pain, pallor, paresthesia, pulselessness, & paralysis]
A. Parethesia (early symptom ni siya)
B. Paralysis
C. Pulselessness (Maam E)
D. Pain (most important and reliable assessment finding to confirm)

64. A 85 year old patient has an accidental fall while going to the bathroom without assistance. It appears the patient has
sustained a bone fracture to the left leg. The leg’s shape is deformed and the patient is unable to move it. The patient is
alert and oriented but in pain. What will you do FIRST after confirming the patient is safe and stable?
A. Immobilize the fracture with a splint.
B. Administer pain medication.
C. Apply an ice pack covered with a towel to the site.
D. Elevate the extremity above heart level.

65. Which statement by a patient, who just received a cast on the right arm for a fracture, requires you to notify the
physician immediately?
A. “I’ve been using ice packs to reduce swelling.””
B. “I can feel my fingers and move them.”
C. “It is really itchy inside my cast!”
D. “My pain is so severe that it hurts to stretch or elevate my arm.”

66. Your patient is 2 hours post-op from a cast placement on the right leg. The patient has family in the room. Which
action by the significant other requires you to re-educate the patient and family about cast care?
A. Positioning the cast at heart level with pillows.
B. Checking the color and temperature of the right foot.
C. Gently moving the cast with the fingertips of the hands every 2 hours to help with drying.
D. Using a hair dryer on the cool setting to help with dwrying.

67. A 20-year-old baseball pitcher has an arthroscopic repair of a rotator cuff injury performed in same-day surgery. When
the nurse plans postoperative teaching for the patient, which information will be included?
A. "Leave the shoulder immobilizer on for the first few days to minimize pain."
B. "You have an appointment with a physical therapist for tomorrow."
C. "The doctor will use the drop-arm test to determine the success of the procedure."
D. "You should try to find a different position to play on the baseball team."

68. During a hearing assessment, the nurse notes that the sound lateralizes to the clients left ear with the Weber test. The
nurse analyzes this result as:
A. A normal finding.
B. A conductive hearing loss in the right ear.
C. A sensorineural or conductive loss.
D. The presence of nystagmus.

You might also like